SlideShare une entreprise Scribd logo
1  sur  148
DIFFUSE PARENCHYMAL LUNG
DISEASE
Dr Nahid Sherbini
Internal Medicine & Pulmonary Consultant
Certified from Harvard Medical School in Practice of clinical Research
LearningTargets -I-
•To elaborate the general diagnostic approach to
ILDs and Classifications
•To diagnose IPF and understand treatment
options
•To recognize forms of IIP and clinical relevance
DPLD I
Idiopathic Interstitial Pneumonias (IIPs)
• IPF
• Other IIPs
• Familial IP
• IP with autoimmune features (IPAF)
• Smoking-related ILDs
LearningTargets -II-
To elaborate the diagnosis to other
ILDs including clinical , radiological
findings and management lines.
DPLD II
• CTD-associated ILDs
•Diffuse Cystic Lung Diseases
Lymphangioleiomyomatosis (LAM)
Pulmonary Langerhans cell histiocytosis (PLCH)
•Others , Pulmonary alveolar proteinosis & DAD
•Radiation Iduced Lung Injury
Introduction
(ILDs) are a heterogeneous group of
disorders that are classified together
because of similar clinical, radiographic,
physiologic, or pathologic manifestations .
Pulmonary interstitium
• Alveolar lining cells
(types 1 and 2)
• Thin elastin-rich
connective
component
containing capillary
blood vessels
What is the Pulmonary Interstitium?
•between the epithelial and
endothelial basement
membrane
•Expansion of the interstitial
compartment by
inflammation with or
without fibrosis
• Necrosis
• Hyperplasia
• Collapse of basement
membrane
• Inflammatory cells
Pathogenesis
The pathogenesis of ILDs is unknown.
But more and more facts have shown that
immune cells and their cytokines play an important
role in the course of ILDs.
Classification
•Divided into
-Associated with known causes and
-Idiopathic.
The treatment choices and prognosis vary among the
different causes and types of ILD
Classification
ILD/DPLD
DPLD of known
Cause
Drugs Exposure
Hypersensitivity
Pneumonitis
Pneumoconiosis
Toxic Inhalation Radiation
CVD
Idiopathic Interstitial
Pneumonias
IPF IIP other than IPF
Desquamative
Interstitial
Pneumonia
Respiratory
Bronchiolitis-
Interstitial Lung
disease
Acute Interstitial
Pneumonia
Cryptogenic
Organizing
Pneumonia
Lymphocytic Interstitial
Pneumonia
Non Specific Interstitial
Pneumonia
Granulomatous
Lung Diseases
(Sarcoidosis)
Others
LAM
Histiocytosis X
Malignancy
IPF: 47-64%
NSIP: 14 to
36%
RBILD/DIP:
10-17%
COP: 4-12%
AIP: 2%
LIP: 2%
Incidence of ILD
Sarcoidosis
8%
Occupation
11% DILD
5% DAH
4%
CTD
9%
Other
11%
Pulmonary Fibrosis
52%
Coultas AJRCCM ; 150:967
(Incidence of IPF=26-31 per 100,000)
To Diagnose ILD
1- Clinical context (clinical features, labs,
PFTs)
2- Disease nature: acute/subacute, chronic
3-Radiologic (HRCT) findings : Distribution
• Pattern of opacities • Associated findings
Q
What is the most important predictor for mortality in IPF patients?
a. Male gender
b. Age
c. DLCO
d. 6MWTS distance
e. Low FVC
Age and Gender
• LAM
Age
Gender
3. Subacute Diseases (weeks to months)
• HSP, Sarcoid, Cellular NSIP, Drug,
“Chronic” EP
__________________________________________________________________________________________________________________
4. Chronic Diseases (months to years)
• UIP, Fibrotic NSIP, Pneumoconioses,
CVD-related, Chronic HSP
Smoking (RBILD and PLCH)
2. Acute Diseases (Days to weeks)
• DAD (AIP), EP, Vasculitis/DPH, Drug, CVD
________________________________________________________________________________________________________________
History: Duration of Illness
Modified Liebow classification of the idiopathic interstitial
pneumonias (Katzenstein)
• Acute
• Acute interstitial pneumonia (AIP)
• Chronic
• Usual interstitial pneumonia /IPF (UIP)
• Subacute
• Nonspecific interstitial pneumonia (NSIP)
• Lymphocytic Interstitial Pneumonia (LIP)
• Cryptogenic Organizing Pneumonia (COP)
• Desquamative interstitial pneumonia (DIP)
• Respiratory bronchiolitis-associated
interstitial lung disease (RBILD)
Revised Classification of IIPs -2013
Major IIPs
• Idiopathic pulmonary fibrosis (idiopathic UIP)
• Idiopathic nonspecific interstitial pneumonia (iNSIP)
• Respiratory bronchiolitis-interstitial lung disease (RB-ILD)
• Desquamative interstitial pneumonia (DIP)
• Cryptogenic organizing pneumonia (COP)
• Acute interstitial pneumonia (AIP)
Rare IIPs
• Idiopathic lymphoid interstitial pneumonia (iLIP)
• Idiopathic pleuroparenchymal fibroelastosis (iPPFE)
Unclassifiable IIP
Physical examinations
•Bilateral basilar, crepitant velcro-like rale
•wheezing, rhonchi and coarse rales are occasionally
heard
•with advanced disease, patients may have tachypnea
and tachycardia
•At last, pulmonary hypertention and cor pulmonale
may be exist
Physical Findings
•RestingTachypnea
•Shallow breathing
•Dry crackles
•Digital clubbing
•Pulmonary HTN
•Non-pulmonary
findings
RADIOLOGICAL
FINDINGS -ILD
Q. 1
Classical HRCT findings for IPF , All true except :
a. Traction Bronchiectasis
b. Basal , Sub pleural , Central
c. Honeycombing
d. Reticular Pattern
Images courtesy of W. Richard Webb, MD.
Basal and peripheral reticulation
Reduced lung volume
nodular
linear
nodular
linear
Honeycomb
Classic IPF HRCT
Image courtesy ofW. RichardWebb, MD.
Reticular opacities Traction
bronchiectasis
Honeycombing
Basal and subpleural predominance
RADIOLOGICAL
FINDINGS
NSIP >30%
Sjogren’s Syndrome
WITH LIP
Am J Respir Crit Care Med. 2000;161:646-664. Slide courtesy of Ganesh Raghu, MD.
DIP
RB-ILD
Peripheral Location
COP IPF
Alveolar Infiltrates
DD
W (Wegner’s)
E (EP)
B (BOOP) COP
A (PAP, Aspiration)
L (Lymphoma)
L (Lipoid Pneumonia)
S (Sacroidosis)
Q. 2
55 year male with history of SOB and dry cough over 6 months. He
smokes occasionally . PFT Restrictive pattern with reduced capacity .
HIS HRCT show GGO with traction bronchiectasis .
Which of the following is the most likely diagnosis?
A. NSIP
B. UIP
C. RB-ILD
D. LIP
Q.3
In interstitial lung diseases, lung function tests most often show:
A. Reduced carbon monoxide diffusing capacity (DLCO)
B. Increased total lung capacity (TLC)
C. Airflow obstruction
D. Elevated arterial PCO2
PFT
•A restrictive defect :
(TLC), (FRC), (RV) ,(FVC) and (FEV1)
but usually the changes are in proportion to the
decreased lung volumes
Low DLCO
PFT
• A reduction (DLCO) is a common, but nonspecific finding in ILD- , the
severity of the DLCO reduction does not correlate well with disease
prognosis, unless the DLCO is less than 35 %.
• Due to effacement of the alveolar capillary units but more importantly, to
the extent of mismatching of ventilation and perfusion of the alveoli.
• In some ILDs, i.e sarcoidosis , there can be considerable reduction in lung
volumes and/or severe hypoxemia but normal or only slightly reduced
DLCO
Moderate to severe reduction of DLCO in the presence of
normal lung volumes in a patient with ILD suggests one
of the following:
a. Combined emphysema and ILD
b. Combined ILD and PVD
c. Pulmonary Langerhans cell histiocytosis
d. Pulmonary lymphangioleiomyomatosis
e. All of the above
Q.4
Q. 5
An interstitial pattern onCXR accompanied by obstructive airflow
suggestive of :
a. Sarcoidosis
b. Diffuse alveolar hemorrhage
c. Pulmonary lymphoangioliomatosis
d. Combined COPD and ILD
e. All of the above
An interstitial pattern on CXR accompanied by obstructive
airflow suggestive of :
• Sarcoidosis
• Lymphangioleiomyomatosis
• HP
• PLCH
• Combined COPD and ILD
6MWT
•6MWT have correlated with prognosis in several studies of IPF
• Pulse oximetry desaturation to 88 during the 6MWT is
associated with a median survival of 3.21 y compared with a
median survival of 6.63 y in those who did not desaturate
below 89%.
Q. 6
Which of the following is false regarding Pulmonary hypertension in
ILDs ?
A.The cause of PH in ILD is likely multifactorial.
b.There is a linear correlation between PFT and PH in ILD.
C. Genetic predisposition not play a role
D. Proposed pathogenesis include presence of vaso-dilation
,angiopathy and PE
E.All are False
The cause of PH in ILD is likely multifactorial
The absence of a linear correlation between PFT
PH in ILD suggests that other factors may play a role ,These include the
following:
1.Vasoconstriction and vascular remodeling;
2.Perivascular fibrosis and vascular destruction;
3.Hypoxemia, both nocturnal and exertional;
4.Thrombotic angiopathy and pulmonary emboli;
5.Elevated pulmonary capillary wedge pressure resulting from peripheral
vascular occlusive disease, which has been described in both IPF and
sarcoidosis and/or diastolic dysfunction;
6.Microvascular inflammation and injury;
7.Pathobiological process (ie, vascular granulomas in sarcoidosis, PH of
Langerhan's cell histiocytosis); and
8.Genetic predisposition and varying gene expression
Q. 7
Which is true about BAL in ILDs ?
a. BAL is less likely to be helpful in patients with a radiographic
pattern suggestive of IPF.
b. BAL does not have an established role in the assessment of ILD
progression or response to therapy
c. Consist normally of macrophages >85%, lymphocytes 10-15%,
neutrophils ≤3%, eosinophils ≤1%, epithelial ≤5%
d. High CD4 /CD8 ratio in sarcoidosis and rheumatoid lung while
reveals low ratio in HP .
e. All are true
Q.
Which of the following associated with neutrophilicCellular pattern
in BAL ?
A. IPF
B. HP
C. COP
D. Drug induced Pneumonitis
E. Radiation Pneumonitis
ROLE OF BRONCHOALVEOLAR
LAVAGE (BAL)
•The lavage fluid is sent for cell counts, cultures for
mycobacterial, viral and fungal pathogens, and cytological
analysis.
•Virtually all patients presenting with hemoptysis and
radiographic ILD should undergo BAL to confirm an alveolar
source of bleeding and identify any infectious etiologies.
BAL
• Normal: macrophages >85%, lymphocytes 10-15%,
neutrophils ≤3%, eosinophils ≤1%, epithelial ≤5%
•Lymphocytic (>15%): sarcoidosis, NSIP, HP,
druginduced, CTDs, radiation, COP, lymphoproliferative
disorders
• Eosinophilic (>1%): eosinophilic pneumonias,
druginduced, BM transplant, asthma, ABPA, infections,
Hodgkin
• Neutrophilic (>3%): CTDs, IPF, aspiration, infections,
bronchitis, asbestosis, ARDS/DAD Bronchoalveolar
Lavage (BAL): Cell Patterns
ATS Guideline.AJRCCM 2012
ILD BAL CD4 CD8T Lymphocyte
Q. 8
Which is true regardingVATS use for diagnosing ILDs ?
A. Low diagnostic accuracy
B. More morbidity and mortality than open lung biopsy
C. Role of BAL andTBBx is highly diagnostic in all IIP
D. Ideal biopsy include two or more surgical wedge biopsies with
areas of normal lung and samples should measure 3-5 cm in length
and 2-3 cm in depth
E. None of the above
Video AssistedThoracic Surgery (VATS)
ChangAC, et al. AnnThorac Surg. 2002.74;1942-1946.
Rena O, et al. Eur JCardiothorac Surg. 1999;16:624-627.
• VATS is the preferred procedure for obtaining a lung biopsy
 High diagnostic accuracy
 Less morbidity and mortality than open lung biopsy
 BAL andTBBx limited to excluding other IPF mimickers
• Ideal biopsy
 Two or more surgical wedge biopsies with areas of normal lung
 Samples should measure 35 cm in length and 23 cm in depth
• Outpatient thoracoscopic lung biopsy can be a safe and effective
procedure for patients with interstitial or focal lung disease
 Diagnosis obtained in 61/62 patients
 72.5 % discharged home within 8 hours
 22.5% discharged home within 23 hours
ATS/ERSConsensus Statement. AmJ Respir Crit Care Med. 2000;161:646-664.
Probability of Histologic Diagnosis of Diffuse Diseases
Surgical
Biopsy
1. Granulomatous diseases
2. Malignant tumors/lymphangitic
3. DAD (any cause)
4. Certain infections
5. Alveolar proteinosis
6. Eosinophilic pneumonia
7.Vasculitis
8. Amyloidosis
9. EG/HX/PLCH
10. LAM
11. RB/RBILD/DIP
12. UIP/NSIP/LIP COP
13. Small airways disease
14. PHT and PVOD
Often
Sometimes
Rare
Transbronchial
Biopsy
Courtesy of Kevin O. Leslie, MD.
Some Pathology Slides
More Slides
Q. 9
• 55 y old , 30 y pack history
• Progressive dyspnea and cough
• Was working in plastic factory for
the last 30 y
• Bilateral infiltrate in chest
radiograph and cyst
• Surgical biopsy shown
What is true about the nature of the
disease ?
a.This stage carry good prognosis
b. Respond to Steroids
c. Changing his work will be
beneficial
d. Poor Prognosis
Q.
• 55 y old , 30 y pack history
• Progressive dyspnea and cough
• Was working in plastic factory for
the last 30 y
• Bilateral infiltrate in chest
radiograph and cyst
• Surgical biopsy shown
What is true about the nature of the
disease ?
a.This stage carry good prognosis
b. Respond to Steroids
c. Changing his work will be
beneficial
d. Poor Prognosis
• Fibrosis with honeycombing
• Architectural destruction
• Peripheral and basal distribution
• Patchy (i.e. normal and abnormal lung)
• Fibroblastic foci UIP
IPF
Average survival diagnosis of IPF is
approximately 2.5–3.5 years1 from diagnosis
Onset of symptoms
Initial visit
Kaplan-Meier plot of the survival
probability in IPF patients (n=238)2
1. Ley B et al.Am J Respir Crit Care Med 2010 October 8 2. KingTE et al. Am J Respir Crit Care Med 2001; 164: 1171-1181
Prevalence IN ME
Q .10
Which of the following conditions cause UIP pattern in
HRCT ?
a. IPF
b. Chronic HP
c. Drug Induced
d. Infections e.gTB
e. All of the above
Establish Diagnosis
Multi-Disciplinary Team (MDT)
Discussion
Clinical
• Symptoms
• Smoking history
• Exposures
• Features of CTD
• Examination
Investigations
• CXR
• CTThorax
• Blood tests
• Lung Function
Pathology
• Bronchoalveolar
lavage
• Surgical lung biopsy
Q.11
A confident diagnosis of idiopathic pulmonary fibrosis
(IPF) requires which one of the following?
A. Surgical lung biopsy
B. Usual interstitial pneumonia (UIP) pattern on lung
biopsy or HRCT.
C. Failure to respond to corticosteroid therapy
D. Evidence of disease progression
To Diagnose
•1. Exclude identifiable causes of ILD (e.g.,
occupational or environmental exposures, drugs
&radiation, CTDs)
•2. UIP pattern shown by: a) HRCT or b) Surgical
lung biopsy, in the absence of HRCT features
inconsistent with UIP Diagnostic Criteria for IPF
(ATS/ERS/JRS/ALAT statement. AJRCCM 2011)
Q. 12
73-year-old retired insulating engineer presents with a 6-m
history of increasing dyspnoea. He worked with asbestos for 2
years, 35 years ago. He has seronegative rheumatoid arthritis,
clubbing and basal crackles on chest examination.The CT scan
is shown below.
Which one of the following is the most likely diagnosis?
a. Idiopathic pulmonary fibrosis
B. Asbestosis
c. Rheumatoid lung
d. Lung adenocarcinoma
e. Bronchiectasis
A. Idiopathic pulmonary fibrosis
• Asbestosis is unlikely because the patient’s asbestos exposure was only
2 years in duration and his disease began more than 20 years later.The
absence of pleural plaques is evidence against asbestosis, in which
more than 95% of patients have pleural plaques demonstrable on chest
CT.
• Rheumatoid lung with interstitial fibrosis is unlikely in seronegative
disease, and clubbing is uncommon in rheumatoid pulmonary fibrosis.
• Lung adenocarcinoma remains a possible diagnosis but in this case is
less likely than IPF and the CT does not suggest the presence of a
cancer.
• Bronchiectasis is unlikely in the absence of cough and sputum
production and clubbing seldom occurs nowadays except in patients
with cystic. Bronchiectasis is not a prominent feature in the presented
CT.
Q 13
• 70 year-old never-smoking man, who is former office worker,
complains of a dry cough and progressive sob (NYHA class III) for
6 m. He takes 20 mg enalapril daily for hypertension. He has no
other diseases. He has not kept animals, or been exposed to dust
or fumes. Auscultation revealsVelcro rales over both lung bases.
There is no clubbing. Pulmonary function tests cannot be
performed because of impressive, possibly psychogenic,
hyperventilation.While breathing room air,ABG shows PaO2 72
mmHg, PaCO2 41 mmHg, pH 7.36 and SaO2 94%. His chest CT
image is shown below.
Q13
What is the initial
diagnostic test ?
a. Serology for CTD
b. VATS
c. BAL
d. TBB
e. Serum precipitating
Ab
Q 14
In patients with suspected idiopathic pulmonary fibrosis, the most
valuable measure is:
A. Bronchoscopy
B. ESR
C.Trial of steroids
D.Video-assisted thorascopic surgery (VATS)
E. None of the above
Q 15
75-year-old female is referred for dyspnoea on exertion and chronic cough
that have worsened progressively over the past 12 months. Pulmonary
function testing reveals an FVC of 72% predicted, FEV1 of 80% predicted
andTLCO of 38% predicted.The chest radiograph shows bilateral interstitial
basal infiltrates. On HRCT, bilateral reticular opacities and clustered basal
honeycombing are found. Open-lung biopsy reveals randomly distributed
foci of usual interstitial pneumonia surrounded by normal lung parenchyma.
What is the most appropriate therapy for this patient?
a. Pirfenidone
b. Bosentan
C. Acetylcysteine
d. Prednisolone/azathioprine
e. Supportive care
The recommendation against the use of the
following agents for the treatment of IPF
• Anticoagulation
• Imatinib
• Combination prednisone, azathioprine, and N-
acetylcysteine
• Selective endothelin receptor antagonist (ambrisentan
• Phosphodiesterase-5 inhibitor (sildenafil)
• Dual endothelin receptor antagonists (macitentan,
bosentan)
(ATS/ERS/JRS/ALAT Guideline.AJRCCM 2015)
Q.
InAscend and CAPACITYTrial ,What is most common reported side
effect of Perfinidone compared to placebo?
A. Vomiting
B. Insomnia
C. Rash
D. Headache
E. Nausea
Q
Pooling the results ofAscend , Capacity 1 and Capacity 2 ,Which is
false ?
a. Improve dyspnoea
b. Reduce mortality
c. Reduce decline in 6 MWT
d. Decrease all cause mortality
e. Reduce decline in FVC
Pirfenidone
ASCEND: Key inclusion criteria
40–80 years of age
Definite UIP on HRCT, or possible UIP on HRCT plus definite or
probable UIP on surgical lung biopsy
Extent of fibrotic changes greater than extent of emphysema on
HRCT scan
FVC ≥50% and ≤90% predicted
DLCO ≥30% and ≤90% predicted
FEV1/FVC ratio ≥0.8
6MWT distance ≥150 m
King TE Jr, et al. N Engl J Med 2014;370:2083-2092.
ASCENDTrial
ASCENDTrial
TOMORROW: Annual rate of decline in
FVC
Difference between nintedanib 150 mg bid and placebo: p=0.064 vs placebo (pre-specified primary multiplicity-corrected
analysis [closed testing procedure]); p=0.014 vs placebo (pre-specified hierarchical testing procedure).
Richeldi L, et al. N Engl J Med 2011;365:1079–1087.
-0.19
-0.17
-0.21
-0.16
-0.06
-0.30
-0.25
-0.20
-0.15
-0.10
-0.05
0.00
Placebo
(n=83)
Nintedanib
50 mg qd
(n=85)
Nintedanib
50 mg bid
(n=86)
Nintedanib
100 mg bid
(n=85)
Nintedanib
150 mg bid
(n=84)
Annual
rate
of
FVC
decline,
L/year
[Mean
(SE)]
TOMORROW: Preservation of health-related
quality of life
5.46 4.67
2.18
1.48
-0.66
-4
-2
0
2
4
6
8
Placebo
(n=79)
Nintedanib
50 mg qd
(n=76)
Nintedanib
50 mg bid
(n=82)
Nintedanib
100 mg bid
(n=82)
Nintedanib
150 mg bid
(n=75)
Change
in
SGRQ
total
score
[Mean
(SE)]
*
*p=0.007 vs placebo.
SGRQ, St George’s Respiratory Questionnaire.
Richeldi L, et al. N Engl J Med 2011;365:1079–1087.
All-cause mortality over 52 weeks: Pooled data
from INPULSIS®
Placebo
Nintedanib 150 mg bid
HR 0.70
(95% CI; 0.43, 1.12)
p=0.1399
Richeldi L, et al. N Engl J Med 2014;370:2071–2082.
Pirfenidone Experience in Saudi
Q 16
About GERD in patients with IPF ,Which of the
following statement correct ?
a. GERD is common (60-90%) in IPF
b. Majority (50-75%) asymptomatic.
c. May contribute to fibrosis progression, AE.
d. Some studies suggest use of GERD medications to be
an independent predictor of longer survival time in IPF,
associated with slower decline in FVC, decreasedAE.
e. All are true
Treatment of IPF
• “We suggest that clinicians use regular antacid
treatment for patients with IPF.”
• “ lung transplantation in patients with IPF.”
• “The committee did not make a recommendation
regarding treatment of PH in patients with IPF.”
(ATS/ERS/JRS/ALAT Guideline. AJRCCM 2015)
Q 17
Which of the following is true regarding NSIP?
A. NSIP is a specific disease entity.
B. Prognosis associated with NSIP and UIP are
similar.
C. NSIP is commonly associated with
connective tissue diseases.
D. NSIP commonly manifests cystic lung lesions
on HRCT.
NSIP EITHER
Idiopathic iNSIP OR Identifiable cause
• Connective tissue diseases
• Drugs
• Environmental/occupational exposures
• Immunocompromised hosts
• Infections
• Resolving acute lung injury
Q 18.
LIP
•idiopathic LIP
•Identifiable cause or underlying disease
Connective tissue disorders – esp. Sjögren
Immunodeficiency
Infections
Drugs/toxins
-Radiologically with GGO ,Cysts
Q 19
• A 50-year-old man, current smoker and HIV with CD4 500, has been complaining
of shortness of breath and non-productive cough for 5 months. He is previously
treated with antibiotics but his symptoms have failed to improve. In the
emergency department, he is noted to be hypoxic on room air and crackles on
auscultation of his lungs. His WBC 16,000; Hgb 14; Plt 300; LDH 500.He had a chest CT
which showed below
• The cell count from the bronchial alveolar lavage reveals eosinophils 5%,
lymphocytes 15%, neutrophils 15%.The transbronchial biopsy shows
inflammatory intraluminal plugs consisting of granulation tissue with fibroblasts
and myofibroblasts in connective matrix, in small airway, ducts and alveoli with
mild interstitial inflammation.There is preservation of architecture and uniform
appearance. What is your presumptive diagnosis?
a. Chronic eosinophilic pneumonia
b. Cryptogenic organizing pneumonia
c. Desquamative interstitial pneumonia
d. Pulmonary Oedema
e. Acute eosinophilic pneumonia
OP
Idiopathic = cryptogenic OP= (idiopathic BOOP) OR
Identifiable cause
• Connective tissue diseases/vasculitides
• Drugs, toxins, radiation
• Infections
• Hypersensitivity pneumonitis
• Aspiration
• Chronic eosinophilic pneumonia
• Diffuse alveolar damage
• Hematologic diseases, allograft transplants
Q 20.
The patient has been discharged on prednisone 20 mg PO daily for 4 weeks.
He has not been compliance to his medications and he comes in
complaining of fatigue and shortness of breath. He is noted to have oxygen
saturation of 95% on room air and afebrile. Repeat radiographs show new
central sparing infiltrates on the left lung.Cultures have been obtained
which has been negative. What would be the next appropriate step?
a. Prednisone 20 mg PO daily
b. Solumedrol 1 g IV daily for 3 days
c. Piperacillin–Tazobactam 3.375 mg IV every 6 hours with vancomycin 1 g
IV every 12 hours
d. Cellcept 1,000 mg PO every 12 hours
e. Amphotericin B
A.
• Answer: A. Prednisone 20 mg PO daily
• The patient appears to have a relapse, which manifests as worsening
symptoms with reoccurrence of prior or new infiltrates.They are
common during steroid taper.
• Predictors of relapse include delayed treatment and mild increases
with alkaline phosphatase and gammaglutamyltransferase.
• Proposed taper of medications include prednisone 0.75 mg/kg/day for 4
weeks; followed by 0.5 mg/kg/day for 4 weeks, and then 20 mg daily
for 4 weeks, 10 mg for 6 weeks, and 5 mg daily for 6 weeks. If relapse
occur while dose <20 mg daily, increase dose to 20 mg daily and slowly
taper accordingly.
• Treatment of COP includes steroids from 0.75 to 1.5 mg/kg/day with
usual duration of up to 1 year.
Q 21.
• 40 y old female , progressive dyspnea for 5 m
• Abnormal CT Fig 1
• TBB reveals adenocarcinoma received cisplatin and gemcitabine EGFR
mutation positive , so started gefitinib . Her symptoms improve
• CT repeated after 2 m of treatment with no new symptoms except sob
Fig 2
• Bronchoscopy done and reveals lymphocytes 30 % CD4/CD8 3/3
• TBB Fig 3 ,Based on results what do you suggest ?
a. Begin Ganciclovir
b. Start radiation
c. Start Antibiotics
d. Stop gefitinib
Fig 1 Fig 2
Fig 3
Q 22
• 40 y old female , progressive dyspnea for 5 m
• Abnormal CT Fig 1
• TBB reveals adenocarcinoma received cisplatin and gemcitabine EGFR
mutation positive , so started gefitinib . Her symptoms improve
• CT repeated after 2 m of treatment Fig 2
• Bronchoscopy done and reveals lymphocytes 30 % CD4/CD8 3/3
• TBB Fig 3 ,Based on results what do you suggest ?
a. Begin Ganciclovir
b. Start radiation
c. Start Antibiotics
d. Stop gefitinib
Q.
• 25 y old lady not known to have
any medical illness , history of
recurrent abortions, presented
with dyspnea , admitted to ICU
CT shown - Bronchoscopy done
Which is true about this
condition?
a. Good prognosis
b. Bronchoscopy will not help in
diagnosis
c. Need high dose steroids
d. Complements will be normal
DAD
• Histologic pattern of ALI characterized by diffuse involvement with
edema, hyaline membranes, and acute interstitial inflammation
(exudative phase) evolving to loose organizing fibrosis and type II
pneumocyte hyperplasia (organizing phase).
• HRCT: Diffuse ground-glass and/or consolidative opacities
• Management: depends on the clinical context, corticosteroids
commonly used when non-infectious
• Prognosis: high short-term mortality
Acute Interstitial Pneumonia
Idiopathic (“Hamman Rich syndrome”)
Identifiable cause or underlying disease:
• Infections
•Toxic inhalants
• Drugs
• CTDs/vasculitides/alveolar hemorrhage
• Acute radiation reaction
• Acute exacerbation in ILDs
Histo – Septal thickening
and proliferation of spindle cells
Q 23
• 40 y old female teacher , Hypothyroidism and hypoadrenalism on
treatment, presented with shortness of breath ,cough a typical chest
pain and haemoptysis- minimal amount. History of Raynaud's and
generalized fatigability .No fever . No other systemic symptoms
• Looks Sick , Fully Oriented , BP 90/60 P130 Afebrile Spo2 90 %
• CVS S1, loud S2 with pansystolic murmur , Chest bilateral crackle and
L L oedema
• Leukopenia ,Mild elevation of transaminase
• CXR show Cardiomegaly and bilateral lung infiltrate
• Previous CT one y back :Interstitial lower lobes infiltrates with traction
bronchiectasis
• Echo Severe Pulmonary HTN 90 and dilated RA ,RV normal LV
• RHC ,ANA Positive ,all other autoimmune profile were negative
Q 23.
What is suggested treatment ?
A. Sildenafil ,Bosentan and Pirfenidone
B. illoprost , Bosentan and steroids
C. Lasix , Steroids ,Sildenafil and illoprost
D. Bosentan , illoprost ,Lasix and Steroids
E. Steroids only
Autoimmune-features ILD
(Interstitial Pneumonia with
Autoimmune Features (IPAF) )
.
Classification criteria:
• Presence of an interstitial pneumonia (by HRCT or SLBx) •
Exclusion of alternative etiologies and,
• Does not meet criteria of a defined connective tissue and, disease
and,
• At least one feature from at least 2/3 domains: clinical (e.g.,
Raynaud’s), serologic, morphologic (HRCT or SLBx features)
ERS/ATS statement. ERJ 2015
RARE ILDS
PPFE
• Idiopathic PPFE OR with Chemotherapy • Hematopoietic stem cell
transplant • Recurrent infections • Familial interstitial pneumonia
• Pleural and subpleural fibrosis with septal elastosis, predominantly
upper lobes
• HRCT: bilateral irregular pleuroparenchymal thickening, more
marked in upper and middle
• Spontaneous pneumomediastinum or pneumothorax common
• no effective treatment identified
• generally poor, most progress
Q. 24
Which is true about Familial Interstitial Pneumonia/ Familial Pulmonary
Fibrosis?
a. >20 % relatives
b. ~40% of interstitial pneumonia / pulmonary fibrosis
c. ~25% of these familial cases have identifiable mutations
d. Spescific HRCT and histopathologic pattern
e. None of the above
Familial Interstitial Pneumonia/
Familial Pulmonary Fibrosis
Evolving recommendations regarding
genetic testing for those with early onset
(<50 yr), positive family history, suspicious
extrapulmonary features
Borie et al. Eur Respir Rev 2017
Q 25
Which one of the following interstitial lung diseases is related to smoking?
A. LAM
B. Desquamative interstitial pneumonia
C. UIP
D. NSIP
E. DAD
Q
Which one of the following interstitial lung diseases is
NOT related to smoking?
A. Acute eosinophilic pneumonia
B. Desquamative interstitial pneumonia
C. Respiratory bronchiolitis –ILD
D. Hypersensitivity Pneumonitis
E. IPF
Smoking-related ILDs
1. Respiratory bronchiolitis-associated ILD (RB-ILD)
2. Desquamative interstitial pneumonia (DIP)
3. Pulmonary Langerhans cell histiocytosis (PLCH)
4. Acute eosinophilic pneumonia (AEP)
5. Combined pulmonary fibrosis and emphysema
(CPFE)
(Vassallo et al. Clin Chest Med 2012)
Q. 26
• 40 y old ,nurse ,15y pack history
• Progressive SOB
• No fever or haemoptysis
• RR 27 SPO2 93%RA
• Chest bilateral crepitation
• Normal Labs including ESR & ANA
• CT Shown
• Best treatment
a. Observation & Stop Smoking
b. Stop Smoking & Steriods
c. Steriods and azathioprin
d. Perfinedone
RB-ILD
• numerous pigmented macrophages
• Relatively uniform appearance
• Most are smoking-related
• HRCT: GGOs ± reticular opacities;
sometimes cysts, or and vague
nodules
• Management: smoking cessation,
corticosteroids
• Prognosis: generally good, up to 30%
mortality
Q 27
• 44 y old came with dyspnea on exertion and cough. Has been told
she has emphysema. Attempt of tobacco cessation failed.
• Physical examination reveals crackles
• Her radiographic ,pathology shown
• Which of the following is most likely ?
a. COPD
b. Goodpasture Syndrome with diffuse pulmonary haemorrhage
c. Pulmonary Langerhans histiocytosis
d. DIP
Q.
Q.
Combined Pulmonary Fibrosis and
Emphysema (CPFE)
•Upper lung emphysema and lower lung fibrosis
•Typically male smokers
•Relatively preserved spirometry and lung volumes with
low DLCO
•Increased incidence of pulmonary hypertension –
associated with increased mortality
•“Pulmonary fibrosis” includes UIP, NSIP, RB-ILD, DIP,
etc.
DPLD II
LearningTargets -II-
To elaborate the diagnosis to other
ILDs including clinical , radiological
findings and management lines.
DPLD II
• CTD-associated ILDs
•Diffuse Cystic Lung Diseases
• Others
Q 28
Respiratory manifestations in CTD is characterized by
which one of the following?
A. Obstructive lung disease is not seen.
B. Lung biopsy is usually needed.
C. SLE involves the lung more often than other CTDs.
D. Acute exacerbation can occur in patients with CTD-
ILDs.
Rheumatic Disease ILD
•RA 20 - 30 %
•PM/DM 20 - 50 %
More common with anti-synthetase antibodies
•Systemic sclerosis 45 % (“clinically significant”)
More common in diffuse disease; topoisomerase-1
antibodies
•SLE 2 - 8 %
Usually in patients with multisystem disease
•MCTD 20 – 60 %
•Sjögren to 25 %
(Castelino et al. Arthritis ResTher 2010)
Q 29
45 year old man known case of PM/DM , presented with three
weeks history of dyspnea on exertion ,progressive and
associated with dry cough ,weight loss and loss of appetite He
has a history of Raynaud’s. Physical Examination show ankle
joint swelling, HRCT show bilateral interstitial infiltrate diffuse
predominantly upper lobes with traction bronchiectasis ILD.
What is true about this disease?
a. 2-5 % of PMDM patients will have it
b. UIP pattern is the commonest to be found in HRCT
c. Anti –jo antibodies positive
d. Obstructive ventilatory defect in PFT
e. None of the above
AntiSythestase Syndrome
•Subset (16-30%) of patients with PM/DM
• Characterized by relatively acute onset, constitutional
symptoms, Raynaud’s, “mechanic’s hands”, arthritis,
ILD.
•anti-Jo-1 (anti-histidyl–tRNA synthetase)
• Associated with ↑ risk of ILD Usually NSIP > UIP > OP;
sometimes LIP, DAD, etc
•Can be more refractory to treat than other PM/DM-
associated ILD
Q 30
• 30 y lady with SLE , Co progressive SOB 3 m , no other respiratory
symptoms . OnWarfarin for previous DVT. Examination is normal
• PFT FEV1 55% ,FVC 58% FEV1FVC 0.78 TLC 68%
RV 100%
DLCO 77% adjusted to alveolar volume 100%
• CXR small lung volume without lung infiltrates or effusion
What is the next to do ?
a. Echo
b. VQ scan
c. Maximum Inspiratory and Expiratory Pressure
d. Bronchoscopy
A.
• MIP ,MEP
• To assess muscle weakness ?myositis or phrenic n palsy
• Dx shrinking lung syndrome
• Tx steroid , theophylline , beta agonist and immunosuppression
Q 31
• 66 y old lady with Systemic sclerosis , Raynaud's
• Never smoker , work as a secretary
• Examination reveals Spo2 93% , diffuse skin thickening and
telangactasia upper limb digitals
• FVC 60%
• HRCT bilateral basal ground glass opacities
What is the best treatment ?
a. Cyclophosamide
b. Cyclosporine
c. Steroids
d. Azathioprine
A.
• B.
• RCT
• 49% improved with cyclophosphamide Vs 26%
Clin Rheumatol. 2006 Mar;25(2):205-12. Epub 2005 Oct 14.
A randomized unblinded trial of cyclophosphamide versus azathioprine in
the treatment of systemic sclerosis.
Nadashkevich O1, Davis P, Fritzler M, Kovalenko W
What is the Diagnosis?
LAM
Bronchiectasis
E
EG
Diffuse Cystic Lung Disease
• Cyst = a round parenchymal lucency with a well defined
thin-wall (<2 mm), usually contain air • Focal/multifocal
vs diffuse
•Cavity = a lucency within pulmonary consolidation, a
mass, or a nodule. (thick wall)
•Emphysema = focal areas of low attenuation without
visible walls
Fleischner Society, 2008
Mechanism of Cyst Formation
•Elastolysis mediated by matrix metalloproteinases
MMPs) – LAM, PLCH
• Destruction of the bronchial wall and progressive
luminal dilatation – PLCH
•Airway narrowing and check valve mechanism – LIP,
amyloidosis
• Hamartoma-like cystic alveolar formation – BHD
• Cavitation of nodule (inflammatory/infectious,
neoplastic)
Q 32
Which of the following radiographic features is
least likely to be found in Langerhans’ cell
histiocytosis of the lung?
a.Nodules ranging in size up to 10 mm
b.Bilateral reticulonodular opacities
c.Pneumothorax
d.Pleural effusion
e.Honeycomb lung
Langerhans’ cell histiocytosis
(LCH)
d. Pleural effusion
1. Early , Centrilobular nodules (2–10 mm in size) and
2. Reticular and nodular opacities with a predominantly bilateral
symmetric upper- to mid-lung distribution.
3. Late ,Cysts develop and become the dominant imaging finding.
Cysts vary in size but usually are smaller than 1 cm may result
in bullous formation, which then predisposes the patient to
recurrent spontaneous pneumothorax. In advanced LCH,
honeycomb changes can occur.
4. Pleural effusions are rare.
Zaveri et al. 2014
Pulmonary Langerhans cell
Histiocytosis
Diagnosis
• BAL: ³5% CD1a cells
• Lung biopsy: bronchoscopic or
VATS
Management
• Stop smoking
• Sometimes, 2-
chlorodeoxyadenosine (2-CdA,
cladribine) •
B-raf inhibitors (e.g.,
vemurafenib)?
• Pulmonary hypertension
• PrognosisVariable, risk of
malignancy
Q 33
• 56 y F , smoker
• Secretary
• Progressive SOB for last 2 y
• Childhood asthma with FH of asthma
• Not on any medications
• Chest examination reduced breath
sounds -No skin lesions
• Previous- 1 y - CXR reported as
increase lung volume.
• FVC 79% FEV1 46% DLCO 60
What is the most likely diagnosis?
a.LAM
b.PLCH
c. Birt-Hogg-Dude syndrome
d.LIP
LAM
• Proliferation of abnormal
smooth muscle cells (LAM cells
– HMB-45+)
• Sporadic andTSC-related
forms; caused by mutation in
theTSC genes
• Mostly women
• High risk of pneumothorax: 60-
80%
• PFT:Typically obstructive
Diagnosis and Management -
LAM
CT chest findings plus any one or more of the following:
• Biopsy of lung or extrapulmonary LAM
• Renal angiomyolipomas
• Chylothorax (seen in 20-40% during course)
•Tuberous sclerosis complex (TSC)
• High serumVEGF-D level, >800 pg/mL Management
What is the name of this radiological
findings?
PAP
• Most are 20-60 y of age (median ~40)
• Nonspecific presentation: insidious onset DOE, cough - sometimes
asymptomatic
• Fever, fatigue, weight loss, chest pain, haemoptysis
• Inspiratory crackles in 20- 50%
• Serum LDH, surfactant A and D, KL-6 (mucinlike glycoprotein) -
common, nonspecific
• Anti-GM-CSF antibodies detectable in serum & BAL fluid in most
cases of acquired PAP
• PFTs: a restrictive defect, reduced DLCO
• Whole Lung Lavage
What is the diagnosis?
Radiation-induced Lung Injury
After radiation therapy in patients with lung cancer and mediastinal
lymphoma, radiologic abnormalities occur in 60-90%; 5-15%
symptomatic.
Radiation pneumonitis - symptoms 4-12 weeks after irradiation
Radiation fibrosis - 6-12 months after irradiation
• Imaging: radiographic abnormalities confined to radiation field
with “straight line” effect
• Management: symptomatic pneumonitis, prednisone 40-60 mg/d x
2 wk, then taper over 4-12 wk
THANKYOU

Contenu connexe

Tendances

Anaesthesia for COPD 15-09-14
Anaesthesia for COPD 15-09-14Anaesthesia for COPD 15-09-14
Anaesthesia for COPD 15-09-14Aftab Hussain
 
Abpa . a diagnostic dilemma
Abpa . a diagnostic dilemmaAbpa . a diagnostic dilemma
Abpa . a diagnostic dilemmaVeerendra Singh
 
Pulmonary alveolar microlithiasis
Pulmonary alveolar microlithiasisPulmonary alveolar microlithiasis
Pulmonary alveolar microlithiasisSesha Sai
 
Hrct chest in interstitial lung diseases
Hrct chest in interstitial lung diseasesHrct chest in interstitial lung diseases
Hrct chest in interstitial lung diseasesAhmed Bahnassy
 
hrct.pptx high resolution ct patterns
hrct.pptx high resolution ct patterns hrct.pptx high resolution ct patterns
hrct.pptx high resolution ct patterns ranjitharadhakrishna3
 
Asthma phenotypes and endotypes
Asthma phenotypes and endotypesAsthma phenotypes and endotypes
Asthma phenotypes and endotypesGamal Agmy
 
Interstitial Lung Disease ( ILD)
Interstitial Lung Disease ( ILD)Interstitial Lung Disease ( ILD)
Interstitial Lung Disease ( ILD)AdityaNag11
 
step by step approach to arterial blood gas analysis
step by step approach to arterial blood gas analysisstep by step approach to arterial blood gas analysis
step by step approach to arterial blood gas analysisikramdr01
 
Pneumomediastinum
PneumomediastinumPneumomediastinum
PneumomediastinumGamal Agmy
 
Asthma-COPD Overlap Syndrome - ACOS
Asthma-COPD Overlap Syndrome - ACOSAsthma-COPD Overlap Syndrome - ACOS
Asthma-COPD Overlap Syndrome - ACOSNino JN Doydora
 
Interstitial lung diseases radiology
Interstitial lung diseases radiologyInterstitial lung diseases radiology
Interstitial lung diseases radiologyShrikant Nagare
 
Occupational Asthma
Occupational AsthmaOccupational Asthma
Occupational AsthmaEneutron
 

Tendances (20)

Anaesthesia for COPD 15-09-14
Anaesthesia for COPD 15-09-14Anaesthesia for COPD 15-09-14
Anaesthesia for COPD 15-09-14
 
Abpa . a diagnostic dilemma
Abpa . a diagnostic dilemmaAbpa . a diagnostic dilemma
Abpa . a diagnostic dilemma
 
Pulmonary alveolar microlithiasis
Pulmonary alveolar microlithiasisPulmonary alveolar microlithiasis
Pulmonary alveolar microlithiasis
 
Hrct chest in interstitial lung diseases
Hrct chest in interstitial lung diseasesHrct chest in interstitial lung diseases
Hrct chest in interstitial lung diseases
 
hrct.pptx high resolution ct patterns
hrct.pptx high resolution ct patterns hrct.pptx high resolution ct patterns
hrct.pptx high resolution ct patterns
 
Asthma phenotypes and endotypes
Asthma phenotypes and endotypesAsthma phenotypes and endotypes
Asthma phenotypes and endotypes
 
Interstitial Lung Disease ( ILD)
Interstitial Lung Disease ( ILD)Interstitial Lung Disease ( ILD)
Interstitial Lung Disease ( ILD)
 
Hypersensitivity pneumonitis
Hypersensitivity pneumonitisHypersensitivity pneumonitis
Hypersensitivity pneumonitis
 
Scleroderma Lung Disease: Best Practices for Monitoring and Treatment
Scleroderma Lung Disease: Best Practices for Monitoring and Treatment Scleroderma Lung Disease: Best Practices for Monitoring and Treatment
Scleroderma Lung Disease: Best Practices for Monitoring and Treatment
 
step by step approach to arterial blood gas analysis
step by step approach to arterial blood gas analysisstep by step approach to arterial blood gas analysis
step by step approach to arterial blood gas analysis
 
Interstitial Lung Disease
Interstitial Lung Disease Interstitial Lung Disease
Interstitial Lung Disease
 
Pneumomediastinum
PneumomediastinumPneumomediastinum
Pneumomediastinum
 
Scleroderma Associated Lung Disease
Scleroderma Associated Lung DiseaseScleroderma Associated Lung Disease
Scleroderma Associated Lung Disease
 
Asthma-COPD Overlap Syndrome - ACOS
Asthma-COPD Overlap Syndrome - ACOSAsthma-COPD Overlap Syndrome - ACOS
Asthma-COPD Overlap Syndrome - ACOS
 
Interstitial lung diseases radiology
Interstitial lung diseases radiologyInterstitial lung diseases radiology
Interstitial lung diseases radiology
 
Occupational Asthma
Occupational AsthmaOccupational Asthma
Occupational Asthma
 
Allergic Bronchopulmonary Aspergillosis
Allergic Bronchopulmonary AspergillosisAllergic Bronchopulmonary Aspergillosis
Allergic Bronchopulmonary Aspergillosis
 
RA-ILD
RA-ILDRA-ILD
RA-ILD
 
Ipf amith
Ipf amithIpf amith
Ipf amith
 
Bronchial Challenge Testing
Bronchial Challenge TestingBronchial Challenge Testing
Bronchial Challenge Testing
 

Similaire à Dpld board reveiw 2019

Dpld board reveiw 2019 final
Dpld board reveiw 2019 finalDpld board reveiw 2019 final
Dpld board reveiw 2019 finalNahid Sherbini
 
Dpld board reveiw final
Dpld board reveiw finalDpld board reveiw final
Dpld board reveiw finalNahid Sherbini
 
2.3.10 Sohal Interstitial Lung Disease.ppt
2.3.10 Sohal Interstitial Lung Disease.ppt2.3.10 Sohal Interstitial Lung Disease.ppt
2.3.10 Sohal Interstitial Lung Disease.pptPankajSharma956210
 
Idiopathic Interstitial Pneumonia With Autoimmune Features(IPAF).pptx
Idiopathic Interstitial Pneumonia With Autoimmune Features(IPAF).pptxIdiopathic Interstitial Pneumonia With Autoimmune Features(IPAF).pptx
Idiopathic Interstitial Pneumonia With Autoimmune Features(IPAF).pptxKefelegnNathan1
 
Connective tissue Disease associated Interstitial Lung Disease
Connective tissue Disease associated Interstitial Lung DiseaseConnective tissue Disease associated Interstitial Lung Disease
Connective tissue Disease associated Interstitial Lung DiseaseOpeyemi Muyiwa
 
interstial lung deases.pptx
interstial lung deases.pptxinterstial lung deases.pptx
interstial lung deases.pptxLway1
 
Tiêu chuẩn chẩn đoán và điều trị copd của ats 1995
Tiêu chuẩn chẩn đoán và điều trị copd của ats 1995Tiêu chuẩn chẩn đoán và điều trị copd của ats 1995
Tiêu chuẩn chẩn đoán và điều trị copd của ats 1995Bệnh Hô Hấp Mãn Tính
 
Pulmonary Hypertension for general physicians
Pulmonary Hypertension for general physicians Pulmonary Hypertension for general physicians
Pulmonary Hypertension for general physicians Sarfraz Saleemi
 
Gold - global initiative against COPD
Gold - global initiative against COPDGold - global initiative against COPD
Gold - global initiative against COPDadithya2115
 
Approach to ild &amp; update
Approach to ild &amp; updateApproach to ild &amp; update
Approach to ild &amp; updateNahid Sherbini
 
Clinical and radiological features of idiopathic interstitial pneumonias (IIP...
Clinical and radiological features of idiopathic interstitial pneumonias (IIP...Clinical and radiological features of idiopathic interstitial pneumonias (IIP...
Clinical and radiological features of idiopathic interstitial pneumonias (IIP...CesarAugustoArroyo
 

Similaire à Dpld board reveiw 2019 (20)

Dpld board reveiw 2019 final
Dpld board reveiw 2019 finalDpld board reveiw 2019 final
Dpld board reveiw 2019 final
 
Dpld board reveiw final
Dpld board reveiw finalDpld board reveiw final
Dpld board reveiw final
 
Ipf forum final
Ipf forum finalIpf forum final
Ipf forum final
 
Interstitial lung disease
Interstitial lung diseaseInterstitial lung disease
Interstitial lung disease
 
Pulmonary Sarcoidosis
Pulmonary SarcoidosisPulmonary Sarcoidosis
Pulmonary Sarcoidosis
 
interstitial lung diseses and idiopathic pulmonary fibrosis
interstitial lung diseses and idiopathic pulmonary fibrosisinterstitial lung diseses and idiopathic pulmonary fibrosis
interstitial lung diseses and idiopathic pulmonary fibrosis
 
Interstitial lung disease
Interstitial lung diseaseInterstitial lung disease
Interstitial lung disease
 
2.3.10 Sohal Interstitial Lung Disease.ppt
2.3.10 Sohal Interstitial Lung Disease.ppt2.3.10 Sohal Interstitial Lung Disease.ppt
2.3.10 Sohal Interstitial Lung Disease.ppt
 
Idiopathic Interstitial Pneumonia With Autoimmune Features(IPAF).pptx
Idiopathic Interstitial Pneumonia With Autoimmune Features(IPAF).pptxIdiopathic Interstitial Pneumonia With Autoimmune Features(IPAF).pptx
Idiopathic Interstitial Pneumonia With Autoimmune Features(IPAF).pptx
 
Interstitial Lung Diseases
Interstitial Lung DiseasesInterstitial Lung Diseases
Interstitial Lung Diseases
 
Connective tissue Disease associated Interstitial Lung Disease
Connective tissue Disease associated Interstitial Lung DiseaseConnective tissue Disease associated Interstitial Lung Disease
Connective tissue Disease associated Interstitial Lung Disease
 
Ild diagnosis
Ild diagnosis Ild diagnosis
Ild diagnosis
 
interstial lung deases.pptx
interstial lung deases.pptxinterstial lung deases.pptx
interstial lung deases.pptx
 
New ulmonary arterial hypertension in rheumatic diseases
New ulmonary arterial hypertension in rheumatic diseases New ulmonary arterial hypertension in rheumatic diseases
New ulmonary arterial hypertension in rheumatic diseases
 
Tiêu chuẩn chẩn đoán và điều trị copd của ats 1995
Tiêu chuẩn chẩn đoán và điều trị copd của ats 1995Tiêu chuẩn chẩn đoán và điều trị copd của ats 1995
Tiêu chuẩn chẩn đoán và điều trị copd của ats 1995
 
ChILD first part
ChILD first partChILD first part
ChILD first part
 
Pulmonary Hypertension for general physicians
Pulmonary Hypertension for general physicians Pulmonary Hypertension for general physicians
Pulmonary Hypertension for general physicians
 
Gold - global initiative against COPD
Gold - global initiative against COPDGold - global initiative against COPD
Gold - global initiative against COPD
 
Approach to ild &amp; update
Approach to ild &amp; updateApproach to ild &amp; update
Approach to ild &amp; update
 
Clinical and radiological features of idiopathic interstitial pneumonias (IIP...
Clinical and radiological features of idiopathic interstitial pneumonias (IIP...Clinical and radiological features of idiopathic interstitial pneumonias (IIP...
Clinical and radiological features of idiopathic interstitial pneumonias (IIP...
 

Plus de Nahid Sherbini

Introduction for scintfic writing
Introduction for scintfic writingIntroduction for scintfic writing
Introduction for scintfic writingNahid Sherbini
 
Respiratory mcq for pulmonary fellowship
Respiratory    mcq for pulmonary fellowshipRespiratory    mcq for pulmonary fellowship
Respiratory mcq for pulmonary fellowshipNahid Sherbini
 
Personalised treatment for asthma
Personalised treatment for asthmaPersonalised treatment for asthma
Personalised treatment for asthmaNahid Sherbini
 
Mers in saudi arabia final
Mers in saudi arabia   finalMers in saudi arabia   final
Mers in saudi arabia finalNahid Sherbini
 
Neuromuscular Disorders Respiratory Complications and Assessment
Neuromuscular Disorders Respiratory Complications and AssessmentNeuromuscular Disorders Respiratory Complications and Assessment
Neuromuscular Disorders Respiratory Complications and AssessmentNahid Sherbini
 
Massive hemoptysis / Nahid Sherbini
Massive hemoptysis / Nahid SherbiniMassive hemoptysis / Nahid Sherbini
Massive hemoptysis / Nahid SherbiniNahid Sherbini
 
The national lung screening trial /Nahid Sherbini
The national lung screening trial /Nahid SherbiniThe national lung screening trial /Nahid Sherbini
The national lung screening trial /Nahid SherbiniNahid Sherbini
 
Osteoporosis هشاشه ناهد
 Osteoporosis هشاشه  ناهد Osteoporosis هشاشه  ناهد
Osteoporosis هشاشه ناهدNahid Sherbini
 
Evaluation of preoperative pulmonary risk By Nahid Sherbini
Evaluation of preoperative pulmonary risk By Nahid SherbiniEvaluation of preoperative pulmonary risk By Nahid Sherbini
Evaluation of preoperative pulmonary risk By Nahid SherbiniNahid Sherbini
 
Evaluation of preoperative pulmonary risk
Evaluation of preoperative pulmonary riskEvaluation of preoperative pulmonary risk
Evaluation of preoperative pulmonary riskNahid Sherbini
 
Control of Ventilation /Lung Physiology by Nahid Sherbini
Control of Ventilation /Lung Physiology by Nahid SherbiniControl of Ventilation /Lung Physiology by Nahid Sherbini
Control of Ventilation /Lung Physiology by Nahid SherbiniNahid Sherbini
 
Asthma in pregnancy \by Nahid Sherbini
Asthma in pregnancy \by Nahid SherbiniAsthma in pregnancy \by Nahid Sherbini
Asthma in pregnancy \by Nahid SherbiniNahid Sherbini
 

Plus de Nahid Sherbini (20)

Reading an article
Reading an articleReading an article
Reading an article
 
Critical Apprasial
Critical Apprasial Critical Apprasial
Critical Apprasial
 
Ebm Nahid Sherbini
Ebm Nahid SherbiniEbm Nahid Sherbini
Ebm Nahid Sherbini
 
Introduction for scintfic writing
Introduction for scintfic writingIntroduction for scintfic writing
Introduction for scintfic writing
 
Respiratory mcq for pulmonary fellowship
Respiratory    mcq for pulmonary fellowshipRespiratory    mcq for pulmonary fellowship
Respiratory mcq for pulmonary fellowship
 
Personalised treatment for asthma
Personalised treatment for asthmaPersonalised treatment for asthma
Personalised treatment for asthma
 
Critical apprasial 2
Critical apprasial 2Critical apprasial 2
Critical apprasial 2
 
Mc qs ild
Mc qs ildMc qs ild
Mc qs ild
 
Pneumonia #
Pneumonia #Pneumonia #
Pneumonia #
 
Mers in saudi arabia final
Mers in saudi arabia   finalMers in saudi arabia   final
Mers in saudi arabia final
 
Neuromuscular Disorders Respiratory Complications and Assessment
Neuromuscular Disorders Respiratory Complications and AssessmentNeuromuscular Disorders Respiratory Complications and Assessment
Neuromuscular Disorders Respiratory Complications and Assessment
 
Massive hemoptysis / Nahid Sherbini
Massive hemoptysis / Nahid SherbiniMassive hemoptysis / Nahid Sherbini
Massive hemoptysis / Nahid Sherbini
 
The national lung screening trial /Nahid Sherbini
The national lung screening trial /Nahid SherbiniThe national lung screening trial /Nahid Sherbini
The national lung screening trial /Nahid Sherbini
 
Osteoporosis هشاشه ناهد
 Osteoporosis هشاشه  ناهد Osteoporosis هشاشه  ناهد
Osteoporosis هشاشه ناهد
 
Evaluation of preoperative pulmonary risk By Nahid Sherbini
Evaluation of preoperative pulmonary risk By Nahid SherbiniEvaluation of preoperative pulmonary risk By Nahid Sherbini
Evaluation of preoperative pulmonary risk By Nahid Sherbini
 
Evaluation of preoperative pulmonary risk
Evaluation of preoperative pulmonary riskEvaluation of preoperative pulmonary risk
Evaluation of preoperative pulmonary risk
 
Control of Ventilation /Lung Physiology by Nahid Sherbini
Control of Ventilation /Lung Physiology by Nahid SherbiniControl of Ventilation /Lung Physiology by Nahid Sherbini
Control of Ventilation /Lung Physiology by Nahid Sherbini
 
Asthma in pregnancy \by Nahid Sherbini
Asthma in pregnancy \by Nahid SherbiniAsthma in pregnancy \by Nahid Sherbini
Asthma in pregnancy \by Nahid Sherbini
 
Control o
Control oControl o
Control o
 
Asthma in pregnancy
Asthma in pregnancyAsthma in pregnancy
Asthma in pregnancy
 

Dernier

Call Girls Tirupati Just Call 9907093804 Top Class Call Girl Service Available
Call Girls Tirupati Just Call 9907093804 Top Class Call Girl Service AvailableCall Girls Tirupati Just Call 9907093804 Top Class Call Girl Service Available
Call Girls Tirupati Just Call 9907093804 Top Class Call Girl Service AvailableDipal Arora
 
Call Girls Dehradun Just Call 9907093804 Top Class Call Girl Service Available
Call Girls Dehradun Just Call 9907093804 Top Class Call Girl Service AvailableCall Girls Dehradun Just Call 9907093804 Top Class Call Girl Service Available
Call Girls Dehradun Just Call 9907093804 Top Class Call Girl Service AvailableDipal Arora
 
Call Girls Jabalpur Just Call 9907093804 Top Class Call Girl Service Available
Call Girls Jabalpur Just Call 9907093804 Top Class Call Girl Service AvailableCall Girls Jabalpur Just Call 9907093804 Top Class Call Girl Service Available
Call Girls Jabalpur Just Call 9907093804 Top Class Call Girl Service AvailableDipal Arora
 
Book Paid Powai Call Girls Mumbai 𖠋 9930245274 𖠋Low Budget Full Independent H...
Book Paid Powai Call Girls Mumbai 𖠋 9930245274 𖠋Low Budget Full Independent H...Book Paid Powai Call Girls Mumbai 𖠋 9930245274 𖠋Low Budget Full Independent H...
Book Paid Powai Call Girls Mumbai 𖠋 9930245274 𖠋Low Budget Full Independent H...Call Girls in Nagpur High Profile
 
Best Rate (Patna ) Call Girls Patna ⟟ 8617370543 ⟟ High Class Call Girl In 5 ...
Best Rate (Patna ) Call Girls Patna ⟟ 8617370543 ⟟ High Class Call Girl In 5 ...Best Rate (Patna ) Call Girls Patna ⟟ 8617370543 ⟟ High Class Call Girl In 5 ...
Best Rate (Patna ) Call Girls Patna ⟟ 8617370543 ⟟ High Class Call Girl In 5 ...Dipal Arora
 
Russian Escorts Girls Nehru Place ZINATHI 🔝9711199012 ☪ 24/7 Call Girls Delhi
Russian Escorts Girls  Nehru Place ZINATHI 🔝9711199012 ☪ 24/7 Call Girls DelhiRussian Escorts Girls  Nehru Place ZINATHI 🔝9711199012 ☪ 24/7 Call Girls Delhi
Russian Escorts Girls Nehru Place ZINATHI 🔝9711199012 ☪ 24/7 Call Girls DelhiAlinaDevecerski
 
Top Rated Bangalore Call Girls Mg Road ⟟ 9332606886 ⟟ Call Me For Genuine S...
Top Rated Bangalore Call Girls Mg Road ⟟   9332606886 ⟟ Call Me For Genuine S...Top Rated Bangalore Call Girls Mg Road ⟟   9332606886 ⟟ Call Me For Genuine S...
Top Rated Bangalore Call Girls Mg Road ⟟ 9332606886 ⟟ Call Me For Genuine S...narwatsonia7
 
The Most Attractive Hyderabad Call Girls Kothapet 𖠋 6297143586 𖠋 Will You Mis...
The Most Attractive Hyderabad Call Girls Kothapet 𖠋 6297143586 𖠋 Will You Mis...The Most Attractive Hyderabad Call Girls Kothapet 𖠋 6297143586 𖠋 Will You Mis...
The Most Attractive Hyderabad Call Girls Kothapet 𖠋 6297143586 𖠋 Will You Mis...chandars293
 
Call Girls Gwalior Just Call 9907093804 Top Class Call Girl Service Available
Call Girls Gwalior Just Call 9907093804 Top Class Call Girl Service AvailableCall Girls Gwalior Just Call 9907093804 Top Class Call Girl Service Available
Call Girls Gwalior Just Call 9907093804 Top Class Call Girl Service AvailableDipal Arora
 
VIP Hyderabad Call Girls Bahadurpally 7877925207 ₹5000 To 25K With AC Room 💚😋
VIP Hyderabad Call Girls Bahadurpally 7877925207 ₹5000 To 25K With AC Room 💚😋VIP Hyderabad Call Girls Bahadurpally 7877925207 ₹5000 To 25K With AC Room 💚😋
VIP Hyderabad Call Girls Bahadurpally 7877925207 ₹5000 To 25K With AC Room 💚😋TANUJA PANDEY
 
Call Girls Aurangabad Just Call 9907093804 Top Class Call Girl Service Available
Call Girls Aurangabad Just Call 9907093804 Top Class Call Girl Service AvailableCall Girls Aurangabad Just Call 9907093804 Top Class Call Girl Service Available
Call Girls Aurangabad Just Call 9907093804 Top Class Call Girl Service AvailableDipal Arora
 
♛VVIP Hyderabad Call Girls Chintalkunta🖕7001035870🖕Riya Kappor Top Call Girl ...
♛VVIP Hyderabad Call Girls Chintalkunta🖕7001035870🖕Riya Kappor Top Call Girl ...♛VVIP Hyderabad Call Girls Chintalkunta🖕7001035870🖕Riya Kappor Top Call Girl ...
♛VVIP Hyderabad Call Girls Chintalkunta🖕7001035870🖕Riya Kappor Top Call Girl ...astropune
 
VIP Call Girls Indore Kirti 💚😋 9256729539 🚀 Indore Escorts
VIP Call Girls Indore Kirti 💚😋  9256729539 🚀 Indore EscortsVIP Call Girls Indore Kirti 💚😋  9256729539 🚀 Indore Escorts
VIP Call Girls Indore Kirti 💚😋 9256729539 🚀 Indore Escortsaditipandeya
 
(👑VVIP ISHAAN ) Russian Call Girls Service Navi Mumbai🖕9920874524🖕Independent...
(👑VVIP ISHAAN ) Russian Call Girls Service Navi Mumbai🖕9920874524🖕Independent...(👑VVIP ISHAAN ) Russian Call Girls Service Navi Mumbai🖕9920874524🖕Independent...
(👑VVIP ISHAAN ) Russian Call Girls Service Navi Mumbai🖕9920874524🖕Independent...Taniya Sharma
 
(Low Rate RASHMI ) Rate Of Call Girls Jaipur ❣ 8445551418 ❣ Elite Models & Ce...
(Low Rate RASHMI ) Rate Of Call Girls Jaipur ❣ 8445551418 ❣ Elite Models & Ce...(Low Rate RASHMI ) Rate Of Call Girls Jaipur ❣ 8445551418 ❣ Elite Models & Ce...
(Low Rate RASHMI ) Rate Of Call Girls Jaipur ❣ 8445551418 ❣ Elite Models & Ce...parulsinha
 
Best Rate (Hyderabad) Call Girls Jahanuma ⟟ 8250192130 ⟟ High Class Call Girl...
Best Rate (Hyderabad) Call Girls Jahanuma ⟟ 8250192130 ⟟ High Class Call Girl...Best Rate (Hyderabad) Call Girls Jahanuma ⟟ 8250192130 ⟟ High Class Call Girl...
Best Rate (Hyderabad) Call Girls Jahanuma ⟟ 8250192130 ⟟ High Class Call Girl...astropune
 
Call Girls Ooty Just Call 9907093804 Top Class Call Girl Service Available
Call Girls Ooty Just Call 9907093804 Top Class Call Girl Service AvailableCall Girls Ooty Just Call 9907093804 Top Class Call Girl Service Available
Call Girls Ooty Just Call 9907093804 Top Class Call Girl Service AvailableDipal Arora
 
Call Girls Bhubaneswar Just Call 9907093804 Top Class Call Girl Service Avail...
Call Girls Bhubaneswar Just Call 9907093804 Top Class Call Girl Service Avail...Call Girls Bhubaneswar Just Call 9907093804 Top Class Call Girl Service Avail...
Call Girls Bhubaneswar Just Call 9907093804 Top Class Call Girl Service Avail...Dipal Arora
 
Call Girls Horamavu WhatsApp Number 7001035870 Meeting With Bangalore Escorts
Call Girls Horamavu WhatsApp Number 7001035870 Meeting With Bangalore EscortsCall Girls Horamavu WhatsApp Number 7001035870 Meeting With Bangalore Escorts
Call Girls Horamavu WhatsApp Number 7001035870 Meeting With Bangalore Escortsvidya singh
 
💎VVIP Kolkata Call Girls Parganas🩱7001035870🩱Independent Girl ( Ac Rooms Avai...
💎VVIP Kolkata Call Girls Parganas🩱7001035870🩱Independent Girl ( Ac Rooms Avai...💎VVIP Kolkata Call Girls Parganas🩱7001035870🩱Independent Girl ( Ac Rooms Avai...
💎VVIP Kolkata Call Girls Parganas🩱7001035870🩱Independent Girl ( Ac Rooms Avai...Taniya Sharma
 

Dernier (20)

Call Girls Tirupati Just Call 9907093804 Top Class Call Girl Service Available
Call Girls Tirupati Just Call 9907093804 Top Class Call Girl Service AvailableCall Girls Tirupati Just Call 9907093804 Top Class Call Girl Service Available
Call Girls Tirupati Just Call 9907093804 Top Class Call Girl Service Available
 
Call Girls Dehradun Just Call 9907093804 Top Class Call Girl Service Available
Call Girls Dehradun Just Call 9907093804 Top Class Call Girl Service AvailableCall Girls Dehradun Just Call 9907093804 Top Class Call Girl Service Available
Call Girls Dehradun Just Call 9907093804 Top Class Call Girl Service Available
 
Call Girls Jabalpur Just Call 9907093804 Top Class Call Girl Service Available
Call Girls Jabalpur Just Call 9907093804 Top Class Call Girl Service AvailableCall Girls Jabalpur Just Call 9907093804 Top Class Call Girl Service Available
Call Girls Jabalpur Just Call 9907093804 Top Class Call Girl Service Available
 
Book Paid Powai Call Girls Mumbai 𖠋 9930245274 𖠋Low Budget Full Independent H...
Book Paid Powai Call Girls Mumbai 𖠋 9930245274 𖠋Low Budget Full Independent H...Book Paid Powai Call Girls Mumbai 𖠋 9930245274 𖠋Low Budget Full Independent H...
Book Paid Powai Call Girls Mumbai 𖠋 9930245274 𖠋Low Budget Full Independent H...
 
Best Rate (Patna ) Call Girls Patna ⟟ 8617370543 ⟟ High Class Call Girl In 5 ...
Best Rate (Patna ) Call Girls Patna ⟟ 8617370543 ⟟ High Class Call Girl In 5 ...Best Rate (Patna ) Call Girls Patna ⟟ 8617370543 ⟟ High Class Call Girl In 5 ...
Best Rate (Patna ) Call Girls Patna ⟟ 8617370543 ⟟ High Class Call Girl In 5 ...
 
Russian Escorts Girls Nehru Place ZINATHI 🔝9711199012 ☪ 24/7 Call Girls Delhi
Russian Escorts Girls  Nehru Place ZINATHI 🔝9711199012 ☪ 24/7 Call Girls DelhiRussian Escorts Girls  Nehru Place ZINATHI 🔝9711199012 ☪ 24/7 Call Girls Delhi
Russian Escorts Girls Nehru Place ZINATHI 🔝9711199012 ☪ 24/7 Call Girls Delhi
 
Top Rated Bangalore Call Girls Mg Road ⟟ 9332606886 ⟟ Call Me For Genuine S...
Top Rated Bangalore Call Girls Mg Road ⟟   9332606886 ⟟ Call Me For Genuine S...Top Rated Bangalore Call Girls Mg Road ⟟   9332606886 ⟟ Call Me For Genuine S...
Top Rated Bangalore Call Girls Mg Road ⟟ 9332606886 ⟟ Call Me For Genuine S...
 
The Most Attractive Hyderabad Call Girls Kothapet 𖠋 6297143586 𖠋 Will You Mis...
The Most Attractive Hyderabad Call Girls Kothapet 𖠋 6297143586 𖠋 Will You Mis...The Most Attractive Hyderabad Call Girls Kothapet 𖠋 6297143586 𖠋 Will You Mis...
The Most Attractive Hyderabad Call Girls Kothapet 𖠋 6297143586 𖠋 Will You Mis...
 
Call Girls Gwalior Just Call 9907093804 Top Class Call Girl Service Available
Call Girls Gwalior Just Call 9907093804 Top Class Call Girl Service AvailableCall Girls Gwalior Just Call 9907093804 Top Class Call Girl Service Available
Call Girls Gwalior Just Call 9907093804 Top Class Call Girl Service Available
 
VIP Hyderabad Call Girls Bahadurpally 7877925207 ₹5000 To 25K With AC Room 💚😋
VIP Hyderabad Call Girls Bahadurpally 7877925207 ₹5000 To 25K With AC Room 💚😋VIP Hyderabad Call Girls Bahadurpally 7877925207 ₹5000 To 25K With AC Room 💚😋
VIP Hyderabad Call Girls Bahadurpally 7877925207 ₹5000 To 25K With AC Room 💚😋
 
Call Girls Aurangabad Just Call 9907093804 Top Class Call Girl Service Available
Call Girls Aurangabad Just Call 9907093804 Top Class Call Girl Service AvailableCall Girls Aurangabad Just Call 9907093804 Top Class Call Girl Service Available
Call Girls Aurangabad Just Call 9907093804 Top Class Call Girl Service Available
 
♛VVIP Hyderabad Call Girls Chintalkunta🖕7001035870🖕Riya Kappor Top Call Girl ...
♛VVIP Hyderabad Call Girls Chintalkunta🖕7001035870🖕Riya Kappor Top Call Girl ...♛VVIP Hyderabad Call Girls Chintalkunta🖕7001035870🖕Riya Kappor Top Call Girl ...
♛VVIP Hyderabad Call Girls Chintalkunta🖕7001035870🖕Riya Kappor Top Call Girl ...
 
VIP Call Girls Indore Kirti 💚😋 9256729539 🚀 Indore Escorts
VIP Call Girls Indore Kirti 💚😋  9256729539 🚀 Indore EscortsVIP Call Girls Indore Kirti 💚😋  9256729539 🚀 Indore Escorts
VIP Call Girls Indore Kirti 💚😋 9256729539 🚀 Indore Escorts
 
(👑VVIP ISHAAN ) Russian Call Girls Service Navi Mumbai🖕9920874524🖕Independent...
(👑VVIP ISHAAN ) Russian Call Girls Service Navi Mumbai🖕9920874524🖕Independent...(👑VVIP ISHAAN ) Russian Call Girls Service Navi Mumbai🖕9920874524🖕Independent...
(👑VVIP ISHAAN ) Russian Call Girls Service Navi Mumbai🖕9920874524🖕Independent...
 
(Low Rate RASHMI ) Rate Of Call Girls Jaipur ❣ 8445551418 ❣ Elite Models & Ce...
(Low Rate RASHMI ) Rate Of Call Girls Jaipur ❣ 8445551418 ❣ Elite Models & Ce...(Low Rate RASHMI ) Rate Of Call Girls Jaipur ❣ 8445551418 ❣ Elite Models & Ce...
(Low Rate RASHMI ) Rate Of Call Girls Jaipur ❣ 8445551418 ❣ Elite Models & Ce...
 
Best Rate (Hyderabad) Call Girls Jahanuma ⟟ 8250192130 ⟟ High Class Call Girl...
Best Rate (Hyderabad) Call Girls Jahanuma ⟟ 8250192130 ⟟ High Class Call Girl...Best Rate (Hyderabad) Call Girls Jahanuma ⟟ 8250192130 ⟟ High Class Call Girl...
Best Rate (Hyderabad) Call Girls Jahanuma ⟟ 8250192130 ⟟ High Class Call Girl...
 
Call Girls Ooty Just Call 9907093804 Top Class Call Girl Service Available
Call Girls Ooty Just Call 9907093804 Top Class Call Girl Service AvailableCall Girls Ooty Just Call 9907093804 Top Class Call Girl Service Available
Call Girls Ooty Just Call 9907093804 Top Class Call Girl Service Available
 
Call Girls Bhubaneswar Just Call 9907093804 Top Class Call Girl Service Avail...
Call Girls Bhubaneswar Just Call 9907093804 Top Class Call Girl Service Avail...Call Girls Bhubaneswar Just Call 9907093804 Top Class Call Girl Service Avail...
Call Girls Bhubaneswar Just Call 9907093804 Top Class Call Girl Service Avail...
 
Call Girls Horamavu WhatsApp Number 7001035870 Meeting With Bangalore Escorts
Call Girls Horamavu WhatsApp Number 7001035870 Meeting With Bangalore EscortsCall Girls Horamavu WhatsApp Number 7001035870 Meeting With Bangalore Escorts
Call Girls Horamavu WhatsApp Number 7001035870 Meeting With Bangalore Escorts
 
💎VVIP Kolkata Call Girls Parganas🩱7001035870🩱Independent Girl ( Ac Rooms Avai...
💎VVIP Kolkata Call Girls Parganas🩱7001035870🩱Independent Girl ( Ac Rooms Avai...💎VVIP Kolkata Call Girls Parganas🩱7001035870🩱Independent Girl ( Ac Rooms Avai...
💎VVIP Kolkata Call Girls Parganas🩱7001035870🩱Independent Girl ( Ac Rooms Avai...
 

Dpld board reveiw 2019

  • 1. DIFFUSE PARENCHYMAL LUNG DISEASE Dr Nahid Sherbini Internal Medicine & Pulmonary Consultant Certified from Harvard Medical School in Practice of clinical Research
  • 2. LearningTargets -I- •To elaborate the general diagnostic approach to ILDs and Classifications •To diagnose IPF and understand treatment options •To recognize forms of IIP and clinical relevance
  • 3. DPLD I Idiopathic Interstitial Pneumonias (IIPs) • IPF • Other IIPs • Familial IP • IP with autoimmune features (IPAF) • Smoking-related ILDs
  • 4. LearningTargets -II- To elaborate the diagnosis to other ILDs including clinical , radiological findings and management lines.
  • 5. DPLD II • CTD-associated ILDs •Diffuse Cystic Lung Diseases Lymphangioleiomyomatosis (LAM) Pulmonary Langerhans cell histiocytosis (PLCH) •Others , Pulmonary alveolar proteinosis & DAD •Radiation Iduced Lung Injury
  • 6. Introduction (ILDs) are a heterogeneous group of disorders that are classified together because of similar clinical, radiographic, physiologic, or pathologic manifestations .
  • 7. Pulmonary interstitium • Alveolar lining cells (types 1 and 2) • Thin elastin-rich connective component containing capillary blood vessels
  • 8. What is the Pulmonary Interstitium? •between the epithelial and endothelial basement membrane •Expansion of the interstitial compartment by inflammation with or without fibrosis • Necrosis • Hyperplasia • Collapse of basement membrane • Inflammatory cells
  • 9. Pathogenesis The pathogenesis of ILDs is unknown. But more and more facts have shown that immune cells and their cytokines play an important role in the course of ILDs.
  • 10. Classification •Divided into -Associated with known causes and -Idiopathic. The treatment choices and prognosis vary among the different causes and types of ILD
  • 12. ILD/DPLD DPLD of known Cause Drugs Exposure Hypersensitivity Pneumonitis Pneumoconiosis Toxic Inhalation Radiation CVD Idiopathic Interstitial Pneumonias IPF IIP other than IPF Desquamative Interstitial Pneumonia Respiratory Bronchiolitis- Interstitial Lung disease Acute Interstitial Pneumonia Cryptogenic Organizing Pneumonia Lymphocytic Interstitial Pneumonia Non Specific Interstitial Pneumonia Granulomatous Lung Diseases (Sarcoidosis) Others LAM Histiocytosis X Malignancy IPF: 47-64% NSIP: 14 to 36% RBILD/DIP: 10-17% COP: 4-12% AIP: 2% LIP: 2%
  • 13. Incidence of ILD Sarcoidosis 8% Occupation 11% DILD 5% DAH 4% CTD 9% Other 11% Pulmonary Fibrosis 52% Coultas AJRCCM ; 150:967 (Incidence of IPF=26-31 per 100,000)
  • 14. To Diagnose ILD 1- Clinical context (clinical features, labs, PFTs) 2- Disease nature: acute/subacute, chronic 3-Radiologic (HRCT) findings : Distribution • Pattern of opacities • Associated findings
  • 15. Q What is the most important predictor for mortality in IPF patients? a. Male gender b. Age c. DLCO d. 6MWTS distance e. Low FVC
  • 16. Age and Gender • LAM Age Gender
  • 17. 3. Subacute Diseases (weeks to months) • HSP, Sarcoid, Cellular NSIP, Drug, “Chronic” EP __________________________________________________________________________________________________________________ 4. Chronic Diseases (months to years) • UIP, Fibrotic NSIP, Pneumoconioses, CVD-related, Chronic HSP Smoking (RBILD and PLCH) 2. Acute Diseases (Days to weeks) • DAD (AIP), EP, Vasculitis/DPH, Drug, CVD ________________________________________________________________________________________________________________ History: Duration of Illness
  • 18. Modified Liebow classification of the idiopathic interstitial pneumonias (Katzenstein) • Acute • Acute interstitial pneumonia (AIP) • Chronic • Usual interstitial pneumonia /IPF (UIP) • Subacute • Nonspecific interstitial pneumonia (NSIP) • Lymphocytic Interstitial Pneumonia (LIP) • Cryptogenic Organizing Pneumonia (COP) • Desquamative interstitial pneumonia (DIP) • Respiratory bronchiolitis-associated interstitial lung disease (RBILD)
  • 19. Revised Classification of IIPs -2013 Major IIPs • Idiopathic pulmonary fibrosis (idiopathic UIP) • Idiopathic nonspecific interstitial pneumonia (iNSIP) • Respiratory bronchiolitis-interstitial lung disease (RB-ILD) • Desquamative interstitial pneumonia (DIP) • Cryptogenic organizing pneumonia (COP) • Acute interstitial pneumonia (AIP) Rare IIPs • Idiopathic lymphoid interstitial pneumonia (iLIP) • Idiopathic pleuroparenchymal fibroelastosis (iPPFE) Unclassifiable IIP
  • 20. Physical examinations •Bilateral basilar, crepitant velcro-like rale •wheezing, rhonchi and coarse rales are occasionally heard •with advanced disease, patients may have tachypnea and tachycardia •At last, pulmonary hypertention and cor pulmonale may be exist
  • 21. Physical Findings •RestingTachypnea •Shallow breathing •Dry crackles •Digital clubbing •Pulmonary HTN •Non-pulmonary findings
  • 22.
  • 23.
  • 24.
  • 26. Q. 1 Classical HRCT findings for IPF , All true except : a. Traction Bronchiectasis b. Basal , Sub pleural , Central c. Honeycombing d. Reticular Pattern
  • 27. Images courtesy of W. Richard Webb, MD. Basal and peripheral reticulation Reduced lung volume
  • 31. Classic IPF HRCT Image courtesy ofW. RichardWebb, MD. Reticular opacities Traction bronchiectasis Honeycombing Basal and subpleural predominance
  • 33. NSIP >30% Sjogren’s Syndrome WITH LIP Am J Respir Crit Care Med. 2000;161:646-664. Slide courtesy of Ganesh Raghu, MD.
  • 34. DIP
  • 38. DD W (Wegner’s) E (EP) B (BOOP) COP A (PAP, Aspiration) L (Lymphoma) L (Lipoid Pneumonia) S (Sacroidosis)
  • 39. Q. 2 55 year male with history of SOB and dry cough over 6 months. He smokes occasionally . PFT Restrictive pattern with reduced capacity . HIS HRCT show GGO with traction bronchiectasis . Which of the following is the most likely diagnosis? A. NSIP B. UIP C. RB-ILD D. LIP
  • 40. Q.3 In interstitial lung diseases, lung function tests most often show: A. Reduced carbon monoxide diffusing capacity (DLCO) B. Increased total lung capacity (TLC) C. Airflow obstruction D. Elevated arterial PCO2
  • 41. PFT •A restrictive defect : (TLC), (FRC), (RV) ,(FVC) and (FEV1) but usually the changes are in proportion to the decreased lung volumes Low DLCO
  • 42. PFT • A reduction (DLCO) is a common, but nonspecific finding in ILD- , the severity of the DLCO reduction does not correlate well with disease prognosis, unless the DLCO is less than 35 %. • Due to effacement of the alveolar capillary units but more importantly, to the extent of mismatching of ventilation and perfusion of the alveoli. • In some ILDs, i.e sarcoidosis , there can be considerable reduction in lung volumes and/or severe hypoxemia but normal or only slightly reduced DLCO
  • 43. Moderate to severe reduction of DLCO in the presence of normal lung volumes in a patient with ILD suggests one of the following: a. Combined emphysema and ILD b. Combined ILD and PVD c. Pulmonary Langerhans cell histiocytosis d. Pulmonary lymphangioleiomyomatosis e. All of the above Q.4
  • 44. Q. 5 An interstitial pattern onCXR accompanied by obstructive airflow suggestive of : a. Sarcoidosis b. Diffuse alveolar hemorrhage c. Pulmonary lymphoangioliomatosis d. Combined COPD and ILD e. All of the above
  • 45. An interstitial pattern on CXR accompanied by obstructive airflow suggestive of : • Sarcoidosis • Lymphangioleiomyomatosis • HP • PLCH • Combined COPD and ILD
  • 46. 6MWT •6MWT have correlated with prognosis in several studies of IPF • Pulse oximetry desaturation to 88 during the 6MWT is associated with a median survival of 3.21 y compared with a median survival of 6.63 y in those who did not desaturate below 89%.
  • 47. Q. 6 Which of the following is false regarding Pulmonary hypertension in ILDs ? A.The cause of PH in ILD is likely multifactorial. b.There is a linear correlation between PFT and PH in ILD. C. Genetic predisposition not play a role D. Proposed pathogenesis include presence of vaso-dilation ,angiopathy and PE E.All are False
  • 48. The cause of PH in ILD is likely multifactorial The absence of a linear correlation between PFT PH in ILD suggests that other factors may play a role ,These include the following: 1.Vasoconstriction and vascular remodeling; 2.Perivascular fibrosis and vascular destruction; 3.Hypoxemia, both nocturnal and exertional; 4.Thrombotic angiopathy and pulmonary emboli; 5.Elevated pulmonary capillary wedge pressure resulting from peripheral vascular occlusive disease, which has been described in both IPF and sarcoidosis and/or diastolic dysfunction; 6.Microvascular inflammation and injury; 7.Pathobiological process (ie, vascular granulomas in sarcoidosis, PH of Langerhan's cell histiocytosis); and 8.Genetic predisposition and varying gene expression
  • 49. Q. 7 Which is true about BAL in ILDs ? a. BAL is less likely to be helpful in patients with a radiographic pattern suggestive of IPF. b. BAL does not have an established role in the assessment of ILD progression or response to therapy c. Consist normally of macrophages >85%, lymphocytes 10-15%, neutrophils ≤3%, eosinophils ≤1%, epithelial ≤5% d. High CD4 /CD8 ratio in sarcoidosis and rheumatoid lung while reveals low ratio in HP . e. All are true
  • 50. Q. Which of the following associated with neutrophilicCellular pattern in BAL ? A. IPF B. HP C. COP D. Drug induced Pneumonitis E. Radiation Pneumonitis
  • 51. ROLE OF BRONCHOALVEOLAR LAVAGE (BAL) •The lavage fluid is sent for cell counts, cultures for mycobacterial, viral and fungal pathogens, and cytological analysis. •Virtually all patients presenting with hemoptysis and radiographic ILD should undergo BAL to confirm an alveolar source of bleeding and identify any infectious etiologies.
  • 52. BAL • Normal: macrophages >85%, lymphocytes 10-15%, neutrophils ≤3%, eosinophils ≤1%, epithelial ≤5% •Lymphocytic (>15%): sarcoidosis, NSIP, HP, druginduced, CTDs, radiation, COP, lymphoproliferative disorders • Eosinophilic (>1%): eosinophilic pneumonias, druginduced, BM transplant, asthma, ABPA, infections, Hodgkin • Neutrophilic (>3%): CTDs, IPF, aspiration, infections, bronchitis, asbestosis, ARDS/DAD Bronchoalveolar Lavage (BAL): Cell Patterns ATS Guideline.AJRCCM 2012
  • 53. ILD BAL CD4 CD8T Lymphocyte
  • 54. Q. 8 Which is true regardingVATS use for diagnosing ILDs ? A. Low diagnostic accuracy B. More morbidity and mortality than open lung biopsy C. Role of BAL andTBBx is highly diagnostic in all IIP D. Ideal biopsy include two or more surgical wedge biopsies with areas of normal lung and samples should measure 3-5 cm in length and 2-3 cm in depth E. None of the above
  • 55. Video AssistedThoracic Surgery (VATS) ChangAC, et al. AnnThorac Surg. 2002.74;1942-1946. Rena O, et al. Eur JCardiothorac Surg. 1999;16:624-627. • VATS is the preferred procedure for obtaining a lung biopsy  High diagnostic accuracy  Less morbidity and mortality than open lung biopsy  BAL andTBBx limited to excluding other IPF mimickers • Ideal biopsy  Two or more surgical wedge biopsies with areas of normal lung  Samples should measure 35 cm in length and 23 cm in depth • Outpatient thoracoscopic lung biopsy can be a safe and effective procedure for patients with interstitial or focal lung disease  Diagnosis obtained in 61/62 patients  72.5 % discharged home within 8 hours  22.5% discharged home within 23 hours ATS/ERSConsensus Statement. AmJ Respir Crit Care Med. 2000;161:646-664.
  • 56. Probability of Histologic Diagnosis of Diffuse Diseases Surgical Biopsy 1. Granulomatous diseases 2. Malignant tumors/lymphangitic 3. DAD (any cause) 4. Certain infections 5. Alveolar proteinosis 6. Eosinophilic pneumonia 7.Vasculitis 8. Amyloidosis 9. EG/HX/PLCH 10. LAM 11. RB/RBILD/DIP 12. UIP/NSIP/LIP COP 13. Small airways disease 14. PHT and PVOD Often Sometimes Rare Transbronchial Biopsy Courtesy of Kevin O. Leslie, MD.
  • 59.
  • 60. Q. 9 • 55 y old , 30 y pack history • Progressive dyspnea and cough • Was working in plastic factory for the last 30 y • Bilateral infiltrate in chest radiograph and cyst • Surgical biopsy shown What is true about the nature of the disease ? a.This stage carry good prognosis b. Respond to Steroids c. Changing his work will be beneficial d. Poor Prognosis
  • 61. Q. • 55 y old , 30 y pack history • Progressive dyspnea and cough • Was working in plastic factory for the last 30 y • Bilateral infiltrate in chest radiograph and cyst • Surgical biopsy shown What is true about the nature of the disease ? a.This stage carry good prognosis b. Respond to Steroids c. Changing his work will be beneficial d. Poor Prognosis • Fibrosis with honeycombing • Architectural destruction • Peripheral and basal distribution • Patchy (i.e. normal and abnormal lung) • Fibroblastic foci UIP
  • 62. IPF
  • 63. Average survival diagnosis of IPF is approximately 2.5–3.5 years1 from diagnosis Onset of symptoms Initial visit Kaplan-Meier plot of the survival probability in IPF patients (n=238)2 1. Ley B et al.Am J Respir Crit Care Med 2010 October 8 2. KingTE et al. Am J Respir Crit Care Med 2001; 164: 1171-1181
  • 65. Q .10 Which of the following conditions cause UIP pattern in HRCT ? a. IPF b. Chronic HP c. Drug Induced d. Infections e.gTB e. All of the above
  • 66. Establish Diagnosis Multi-Disciplinary Team (MDT) Discussion Clinical • Symptoms • Smoking history • Exposures • Features of CTD • Examination Investigations • CXR • CTThorax • Blood tests • Lung Function Pathology • Bronchoalveolar lavage • Surgical lung biopsy
  • 67. Q.11 A confident diagnosis of idiopathic pulmonary fibrosis (IPF) requires which one of the following? A. Surgical lung biopsy B. Usual interstitial pneumonia (UIP) pattern on lung biopsy or HRCT. C. Failure to respond to corticosteroid therapy D. Evidence of disease progression
  • 68. To Diagnose •1. Exclude identifiable causes of ILD (e.g., occupational or environmental exposures, drugs &radiation, CTDs) •2. UIP pattern shown by: a) HRCT or b) Surgical lung biopsy, in the absence of HRCT features inconsistent with UIP Diagnostic Criteria for IPF (ATS/ERS/JRS/ALAT statement. AJRCCM 2011)
  • 69. Q. 12 73-year-old retired insulating engineer presents with a 6-m history of increasing dyspnoea. He worked with asbestos for 2 years, 35 years ago. He has seronegative rheumatoid arthritis, clubbing and basal crackles on chest examination.The CT scan is shown below. Which one of the following is the most likely diagnosis? a. Idiopathic pulmonary fibrosis B. Asbestosis c. Rheumatoid lung d. Lung adenocarcinoma e. Bronchiectasis
  • 70. A. Idiopathic pulmonary fibrosis • Asbestosis is unlikely because the patient’s asbestos exposure was only 2 years in duration and his disease began more than 20 years later.The absence of pleural plaques is evidence against asbestosis, in which more than 95% of patients have pleural plaques demonstrable on chest CT. • Rheumatoid lung with interstitial fibrosis is unlikely in seronegative disease, and clubbing is uncommon in rheumatoid pulmonary fibrosis. • Lung adenocarcinoma remains a possible diagnosis but in this case is less likely than IPF and the CT does not suggest the presence of a cancer. • Bronchiectasis is unlikely in the absence of cough and sputum production and clubbing seldom occurs nowadays except in patients with cystic. Bronchiectasis is not a prominent feature in the presented CT.
  • 71. Q 13 • 70 year-old never-smoking man, who is former office worker, complains of a dry cough and progressive sob (NYHA class III) for 6 m. He takes 20 mg enalapril daily for hypertension. He has no other diseases. He has not kept animals, or been exposed to dust or fumes. Auscultation revealsVelcro rales over both lung bases. There is no clubbing. Pulmonary function tests cannot be performed because of impressive, possibly psychogenic, hyperventilation.While breathing room air,ABG shows PaO2 72 mmHg, PaCO2 41 mmHg, pH 7.36 and SaO2 94%. His chest CT image is shown below.
  • 72. Q13 What is the initial diagnostic test ? a. Serology for CTD b. VATS c. BAL d. TBB e. Serum precipitating Ab
  • 73. Q 14 In patients with suspected idiopathic pulmonary fibrosis, the most valuable measure is: A. Bronchoscopy B. ESR C.Trial of steroids D.Video-assisted thorascopic surgery (VATS) E. None of the above
  • 74. Q 15 75-year-old female is referred for dyspnoea on exertion and chronic cough that have worsened progressively over the past 12 months. Pulmonary function testing reveals an FVC of 72% predicted, FEV1 of 80% predicted andTLCO of 38% predicted.The chest radiograph shows bilateral interstitial basal infiltrates. On HRCT, bilateral reticular opacities and clustered basal honeycombing are found. Open-lung biopsy reveals randomly distributed foci of usual interstitial pneumonia surrounded by normal lung parenchyma. What is the most appropriate therapy for this patient? a. Pirfenidone b. Bosentan C. Acetylcysteine d. Prednisolone/azathioprine e. Supportive care
  • 75.
  • 76. The recommendation against the use of the following agents for the treatment of IPF • Anticoagulation • Imatinib • Combination prednisone, azathioprine, and N- acetylcysteine • Selective endothelin receptor antagonist (ambrisentan • Phosphodiesterase-5 inhibitor (sildenafil) • Dual endothelin receptor antagonists (macitentan, bosentan) (ATS/ERS/JRS/ALAT Guideline.AJRCCM 2015)
  • 77. Q. InAscend and CAPACITYTrial ,What is most common reported side effect of Perfinidone compared to placebo? A. Vomiting B. Insomnia C. Rash D. Headache E. Nausea
  • 78. Q Pooling the results ofAscend , Capacity 1 and Capacity 2 ,Which is false ? a. Improve dyspnoea b. Reduce mortality c. Reduce decline in 6 MWT d. Decrease all cause mortality e. Reduce decline in FVC
  • 79.
  • 81. ASCEND: Key inclusion criteria 40–80 years of age Definite UIP on HRCT, or possible UIP on HRCT plus definite or probable UIP on surgical lung biopsy Extent of fibrotic changes greater than extent of emphysema on HRCT scan FVC ≥50% and ≤90% predicted DLCO ≥30% and ≤90% predicted FEV1/FVC ratio ≥0.8 6MWT distance ≥150 m King TE Jr, et al. N Engl J Med 2014;370:2083-2092.
  • 84. TOMORROW: Annual rate of decline in FVC Difference between nintedanib 150 mg bid and placebo: p=0.064 vs placebo (pre-specified primary multiplicity-corrected analysis [closed testing procedure]); p=0.014 vs placebo (pre-specified hierarchical testing procedure). Richeldi L, et al. N Engl J Med 2011;365:1079–1087. -0.19 -0.17 -0.21 -0.16 -0.06 -0.30 -0.25 -0.20 -0.15 -0.10 -0.05 0.00 Placebo (n=83) Nintedanib 50 mg qd (n=85) Nintedanib 50 mg bid (n=86) Nintedanib 100 mg bid (n=85) Nintedanib 150 mg bid (n=84) Annual rate of FVC decline, L/year [Mean (SE)]
  • 85. TOMORROW: Preservation of health-related quality of life 5.46 4.67 2.18 1.48 -0.66 -4 -2 0 2 4 6 8 Placebo (n=79) Nintedanib 50 mg qd (n=76) Nintedanib 50 mg bid (n=82) Nintedanib 100 mg bid (n=82) Nintedanib 150 mg bid (n=75) Change in SGRQ total score [Mean (SE)] * *p=0.007 vs placebo. SGRQ, St George’s Respiratory Questionnaire. Richeldi L, et al. N Engl J Med 2011;365:1079–1087.
  • 86.
  • 87. All-cause mortality over 52 weeks: Pooled data from INPULSIS® Placebo Nintedanib 150 mg bid HR 0.70 (95% CI; 0.43, 1.12) p=0.1399 Richeldi L, et al. N Engl J Med 2014;370:2071–2082.
  • 89. Q 16 About GERD in patients with IPF ,Which of the following statement correct ? a. GERD is common (60-90%) in IPF b. Majority (50-75%) asymptomatic. c. May contribute to fibrosis progression, AE. d. Some studies suggest use of GERD medications to be an independent predictor of longer survival time in IPF, associated with slower decline in FVC, decreasedAE. e. All are true
  • 90. Treatment of IPF • “We suggest that clinicians use regular antacid treatment for patients with IPF.” • “ lung transplantation in patients with IPF.” • “The committee did not make a recommendation regarding treatment of PH in patients with IPF.” (ATS/ERS/JRS/ALAT Guideline. AJRCCM 2015)
  • 91. Q 17 Which of the following is true regarding NSIP? A. NSIP is a specific disease entity. B. Prognosis associated with NSIP and UIP are similar. C. NSIP is commonly associated with connective tissue diseases. D. NSIP commonly manifests cystic lung lesions on HRCT.
  • 92. NSIP EITHER Idiopathic iNSIP OR Identifiable cause • Connective tissue diseases • Drugs • Environmental/occupational exposures • Immunocompromised hosts • Infections • Resolving acute lung injury
  • 93. Q 18.
  • 94.
  • 95. LIP •idiopathic LIP •Identifiable cause or underlying disease Connective tissue disorders – esp. Sjögren Immunodeficiency Infections Drugs/toxins -Radiologically with GGO ,Cysts
  • 96. Q 19 • A 50-year-old man, current smoker and HIV with CD4 500, has been complaining of shortness of breath and non-productive cough for 5 months. He is previously treated with antibiotics but his symptoms have failed to improve. In the emergency department, he is noted to be hypoxic on room air and crackles on auscultation of his lungs. His WBC 16,000; Hgb 14; Plt 300; LDH 500.He had a chest CT which showed below • The cell count from the bronchial alveolar lavage reveals eosinophils 5%, lymphocytes 15%, neutrophils 15%.The transbronchial biopsy shows inflammatory intraluminal plugs consisting of granulation tissue with fibroblasts and myofibroblasts in connective matrix, in small airway, ducts and alveoli with mild interstitial inflammation.There is preservation of architecture and uniform appearance. What is your presumptive diagnosis? a. Chronic eosinophilic pneumonia b. Cryptogenic organizing pneumonia c. Desquamative interstitial pneumonia d. Pulmonary Oedema e. Acute eosinophilic pneumonia
  • 97. OP Idiopathic = cryptogenic OP= (idiopathic BOOP) OR Identifiable cause • Connective tissue diseases/vasculitides • Drugs, toxins, radiation • Infections • Hypersensitivity pneumonitis • Aspiration • Chronic eosinophilic pneumonia • Diffuse alveolar damage • Hematologic diseases, allograft transplants
  • 98. Q 20. The patient has been discharged on prednisone 20 mg PO daily for 4 weeks. He has not been compliance to his medications and he comes in complaining of fatigue and shortness of breath. He is noted to have oxygen saturation of 95% on room air and afebrile. Repeat radiographs show new central sparing infiltrates on the left lung.Cultures have been obtained which has been negative. What would be the next appropriate step? a. Prednisone 20 mg PO daily b. Solumedrol 1 g IV daily for 3 days c. Piperacillin–Tazobactam 3.375 mg IV every 6 hours with vancomycin 1 g IV every 12 hours d. Cellcept 1,000 mg PO every 12 hours e. Amphotericin B
  • 99. A. • Answer: A. Prednisone 20 mg PO daily • The patient appears to have a relapse, which manifests as worsening symptoms with reoccurrence of prior or new infiltrates.They are common during steroid taper. • Predictors of relapse include delayed treatment and mild increases with alkaline phosphatase and gammaglutamyltransferase. • Proposed taper of medications include prednisone 0.75 mg/kg/day for 4 weeks; followed by 0.5 mg/kg/day for 4 weeks, and then 20 mg daily for 4 weeks, 10 mg for 6 weeks, and 5 mg daily for 6 weeks. If relapse occur while dose <20 mg daily, increase dose to 20 mg daily and slowly taper accordingly. • Treatment of COP includes steroids from 0.75 to 1.5 mg/kg/day with usual duration of up to 1 year.
  • 100. Q 21. • 40 y old female , progressive dyspnea for 5 m • Abnormal CT Fig 1 • TBB reveals adenocarcinoma received cisplatin and gemcitabine EGFR mutation positive , so started gefitinib . Her symptoms improve • CT repeated after 2 m of treatment with no new symptoms except sob Fig 2 • Bronchoscopy done and reveals lymphocytes 30 % CD4/CD8 3/3 • TBB Fig 3 ,Based on results what do you suggest ? a. Begin Ganciclovir b. Start radiation c. Start Antibiotics d. Stop gefitinib
  • 101. Fig 1 Fig 2
  • 102. Fig 3
  • 103. Q 22 • 40 y old female , progressive dyspnea for 5 m • Abnormal CT Fig 1 • TBB reveals adenocarcinoma received cisplatin and gemcitabine EGFR mutation positive , so started gefitinib . Her symptoms improve • CT repeated after 2 m of treatment Fig 2 • Bronchoscopy done and reveals lymphocytes 30 % CD4/CD8 3/3 • TBB Fig 3 ,Based on results what do you suggest ? a. Begin Ganciclovir b. Start radiation c. Start Antibiotics d. Stop gefitinib
  • 104. Q. • 25 y old lady not known to have any medical illness , history of recurrent abortions, presented with dyspnea , admitted to ICU CT shown - Bronchoscopy done Which is true about this condition? a. Good prognosis b. Bronchoscopy will not help in diagnosis c. Need high dose steroids d. Complements will be normal
  • 105. DAD • Histologic pattern of ALI characterized by diffuse involvement with edema, hyaline membranes, and acute interstitial inflammation (exudative phase) evolving to loose organizing fibrosis and type II pneumocyte hyperplasia (organizing phase). • HRCT: Diffuse ground-glass and/or consolidative opacities • Management: depends on the clinical context, corticosteroids commonly used when non-infectious • Prognosis: high short-term mortality
  • 106. Acute Interstitial Pneumonia Idiopathic (“Hamman Rich syndrome”) Identifiable cause or underlying disease: • Infections •Toxic inhalants • Drugs • CTDs/vasculitides/alveolar hemorrhage • Acute radiation reaction • Acute exacerbation in ILDs Histo – Septal thickening and proliferation of spindle cells
  • 107. Q 23 • 40 y old female teacher , Hypothyroidism and hypoadrenalism on treatment, presented with shortness of breath ,cough a typical chest pain and haemoptysis- minimal amount. History of Raynaud's and generalized fatigability .No fever . No other systemic symptoms • Looks Sick , Fully Oriented , BP 90/60 P130 Afebrile Spo2 90 % • CVS S1, loud S2 with pansystolic murmur , Chest bilateral crackle and L L oedema • Leukopenia ,Mild elevation of transaminase • CXR show Cardiomegaly and bilateral lung infiltrate • Previous CT one y back :Interstitial lower lobes infiltrates with traction bronchiectasis • Echo Severe Pulmonary HTN 90 and dilated RA ,RV normal LV • RHC ,ANA Positive ,all other autoimmune profile were negative
  • 108. Q 23. What is suggested treatment ? A. Sildenafil ,Bosentan and Pirfenidone B. illoprost , Bosentan and steroids C. Lasix , Steroids ,Sildenafil and illoprost D. Bosentan , illoprost ,Lasix and Steroids E. Steroids only
  • 109. Autoimmune-features ILD (Interstitial Pneumonia with Autoimmune Features (IPAF) ) . Classification criteria: • Presence of an interstitial pneumonia (by HRCT or SLBx) • Exclusion of alternative etiologies and, • Does not meet criteria of a defined connective tissue and, disease and, • At least one feature from at least 2/3 domains: clinical (e.g., Raynaud’s), serologic, morphologic (HRCT or SLBx features) ERS/ATS statement. ERJ 2015
  • 111. PPFE • Idiopathic PPFE OR with Chemotherapy • Hematopoietic stem cell transplant • Recurrent infections • Familial interstitial pneumonia • Pleural and subpleural fibrosis with septal elastosis, predominantly upper lobes • HRCT: bilateral irregular pleuroparenchymal thickening, more marked in upper and middle • Spontaneous pneumomediastinum or pneumothorax common • no effective treatment identified • generally poor, most progress
  • 112. Q. 24 Which is true about Familial Interstitial Pneumonia/ Familial Pulmonary Fibrosis? a. >20 % relatives b. ~40% of interstitial pneumonia / pulmonary fibrosis c. ~25% of these familial cases have identifiable mutations d. Spescific HRCT and histopathologic pattern e. None of the above
  • 113. Familial Interstitial Pneumonia/ Familial Pulmonary Fibrosis Evolving recommendations regarding genetic testing for those with early onset (<50 yr), positive family history, suspicious extrapulmonary features Borie et al. Eur Respir Rev 2017
  • 114. Q 25 Which one of the following interstitial lung diseases is related to smoking? A. LAM B. Desquamative interstitial pneumonia C. UIP D. NSIP E. DAD
  • 115. Q Which one of the following interstitial lung diseases is NOT related to smoking? A. Acute eosinophilic pneumonia B. Desquamative interstitial pneumonia C. Respiratory bronchiolitis –ILD D. Hypersensitivity Pneumonitis E. IPF
  • 116. Smoking-related ILDs 1. Respiratory bronchiolitis-associated ILD (RB-ILD) 2. Desquamative interstitial pneumonia (DIP) 3. Pulmonary Langerhans cell histiocytosis (PLCH) 4. Acute eosinophilic pneumonia (AEP) 5. Combined pulmonary fibrosis and emphysema (CPFE) (Vassallo et al. Clin Chest Med 2012)
  • 117. Q. 26 • 40 y old ,nurse ,15y pack history • Progressive SOB • No fever or haemoptysis • RR 27 SPO2 93%RA • Chest bilateral crepitation • Normal Labs including ESR & ANA • CT Shown • Best treatment a. Observation & Stop Smoking b. Stop Smoking & Steriods c. Steriods and azathioprin d. Perfinedone
  • 118. RB-ILD • numerous pigmented macrophages • Relatively uniform appearance • Most are smoking-related • HRCT: GGOs ± reticular opacities; sometimes cysts, or and vague nodules • Management: smoking cessation, corticosteroids • Prognosis: generally good, up to 30% mortality
  • 119. Q 27 • 44 y old came with dyspnea on exertion and cough. Has been told she has emphysema. Attempt of tobacco cessation failed. • Physical examination reveals crackles • Her radiographic ,pathology shown • Which of the following is most likely ? a. COPD b. Goodpasture Syndrome with diffuse pulmonary haemorrhage c. Pulmonary Langerhans histiocytosis d. DIP
  • 120. Q.
  • 121. Q.
  • 122. Combined Pulmonary Fibrosis and Emphysema (CPFE) •Upper lung emphysema and lower lung fibrosis •Typically male smokers •Relatively preserved spirometry and lung volumes with low DLCO •Increased incidence of pulmonary hypertension – associated with increased mortality •“Pulmonary fibrosis” includes UIP, NSIP, RB-ILD, DIP, etc.
  • 124. LearningTargets -II- To elaborate the diagnosis to other ILDs including clinical , radiological findings and management lines.
  • 125. DPLD II • CTD-associated ILDs •Diffuse Cystic Lung Diseases • Others
  • 126. Q 28 Respiratory manifestations in CTD is characterized by which one of the following? A. Obstructive lung disease is not seen. B. Lung biopsy is usually needed. C. SLE involves the lung more often than other CTDs. D. Acute exacerbation can occur in patients with CTD- ILDs.
  • 127. Rheumatic Disease ILD •RA 20 - 30 % •PM/DM 20 - 50 % More common with anti-synthetase antibodies •Systemic sclerosis 45 % (“clinically significant”) More common in diffuse disease; topoisomerase-1 antibodies •SLE 2 - 8 % Usually in patients with multisystem disease •MCTD 20 – 60 % •Sjögren to 25 % (Castelino et al. Arthritis ResTher 2010)
  • 128. Q 29 45 year old man known case of PM/DM , presented with three weeks history of dyspnea on exertion ,progressive and associated with dry cough ,weight loss and loss of appetite He has a history of Raynaud’s. Physical Examination show ankle joint swelling, HRCT show bilateral interstitial infiltrate diffuse predominantly upper lobes with traction bronchiectasis ILD. What is true about this disease? a. 2-5 % of PMDM patients will have it b. UIP pattern is the commonest to be found in HRCT c. Anti –jo antibodies positive d. Obstructive ventilatory defect in PFT e. None of the above
  • 129. AntiSythestase Syndrome •Subset (16-30%) of patients with PM/DM • Characterized by relatively acute onset, constitutional symptoms, Raynaud’s, “mechanic’s hands”, arthritis, ILD. •anti-Jo-1 (anti-histidyl–tRNA synthetase) • Associated with ↑ risk of ILD Usually NSIP > UIP > OP; sometimes LIP, DAD, etc •Can be more refractory to treat than other PM/DM- associated ILD
  • 130. Q 30 • 30 y lady with SLE , Co progressive SOB 3 m , no other respiratory symptoms . OnWarfarin for previous DVT. Examination is normal • PFT FEV1 55% ,FVC 58% FEV1FVC 0.78 TLC 68% RV 100% DLCO 77% adjusted to alveolar volume 100% • CXR small lung volume without lung infiltrates or effusion What is the next to do ? a. Echo b. VQ scan c. Maximum Inspiratory and Expiratory Pressure d. Bronchoscopy
  • 131. A. • MIP ,MEP • To assess muscle weakness ?myositis or phrenic n palsy • Dx shrinking lung syndrome • Tx steroid , theophylline , beta agonist and immunosuppression
  • 132. Q 31 • 66 y old lady with Systemic sclerosis , Raynaud's • Never smoker , work as a secretary • Examination reveals Spo2 93% , diffuse skin thickening and telangactasia upper limb digitals • FVC 60% • HRCT bilateral basal ground glass opacities What is the best treatment ? a. Cyclophosamide b. Cyclosporine c. Steroids d. Azathioprine
  • 133. A. • B. • RCT • 49% improved with cyclophosphamide Vs 26% Clin Rheumatol. 2006 Mar;25(2):205-12. Epub 2005 Oct 14. A randomized unblinded trial of cyclophosphamide versus azathioprine in the treatment of systemic sclerosis. Nadashkevich O1, Davis P, Fritzler M, Kovalenko W
  • 134. What is the Diagnosis? LAM Bronchiectasis E EG
  • 135. Diffuse Cystic Lung Disease • Cyst = a round parenchymal lucency with a well defined thin-wall (<2 mm), usually contain air • Focal/multifocal vs diffuse •Cavity = a lucency within pulmonary consolidation, a mass, or a nodule. (thick wall) •Emphysema = focal areas of low attenuation without visible walls Fleischner Society, 2008
  • 136. Mechanism of Cyst Formation •Elastolysis mediated by matrix metalloproteinases MMPs) – LAM, PLCH • Destruction of the bronchial wall and progressive luminal dilatation – PLCH •Airway narrowing and check valve mechanism – LIP, amyloidosis • Hamartoma-like cystic alveolar formation – BHD • Cavitation of nodule (inflammatory/infectious, neoplastic)
  • 137. Q 32 Which of the following radiographic features is least likely to be found in Langerhans’ cell histiocytosis of the lung? a.Nodules ranging in size up to 10 mm b.Bilateral reticulonodular opacities c.Pneumothorax d.Pleural effusion e.Honeycomb lung
  • 138.
  • 139. Langerhans’ cell histiocytosis (LCH) d. Pleural effusion 1. Early , Centrilobular nodules (2–10 mm in size) and 2. Reticular and nodular opacities with a predominantly bilateral symmetric upper- to mid-lung distribution. 3. Late ,Cysts develop and become the dominant imaging finding. Cysts vary in size but usually are smaller than 1 cm may result in bullous formation, which then predisposes the patient to recurrent spontaneous pneumothorax. In advanced LCH, honeycomb changes can occur. 4. Pleural effusions are rare. Zaveri et al. 2014
  • 140. Pulmonary Langerhans cell Histiocytosis Diagnosis • BAL: ³5% CD1a cells • Lung biopsy: bronchoscopic or VATS Management • Stop smoking • Sometimes, 2- chlorodeoxyadenosine (2-CdA, cladribine) • B-raf inhibitors (e.g., vemurafenib)? • Pulmonary hypertension • PrognosisVariable, risk of malignancy
  • 141. Q 33 • 56 y F , smoker • Secretary • Progressive SOB for last 2 y • Childhood asthma with FH of asthma • Not on any medications • Chest examination reduced breath sounds -No skin lesions • Previous- 1 y - CXR reported as increase lung volume. • FVC 79% FEV1 46% DLCO 60 What is the most likely diagnosis? a.LAM b.PLCH c. Birt-Hogg-Dude syndrome d.LIP
  • 142. LAM • Proliferation of abnormal smooth muscle cells (LAM cells – HMB-45+) • Sporadic andTSC-related forms; caused by mutation in theTSC genes • Mostly women • High risk of pneumothorax: 60- 80% • PFT:Typically obstructive
  • 143. Diagnosis and Management - LAM CT chest findings plus any one or more of the following: • Biopsy of lung or extrapulmonary LAM • Renal angiomyolipomas • Chylothorax (seen in 20-40% during course) •Tuberous sclerosis complex (TSC) • High serumVEGF-D level, >800 pg/mL Management
  • 144. What is the name of this radiological findings?
  • 145. PAP • Most are 20-60 y of age (median ~40) • Nonspecific presentation: insidious onset DOE, cough - sometimes asymptomatic • Fever, fatigue, weight loss, chest pain, haemoptysis • Inspiratory crackles in 20- 50% • Serum LDH, surfactant A and D, KL-6 (mucinlike glycoprotein) - common, nonspecific • Anti-GM-CSF antibodies detectable in serum & BAL fluid in most cases of acquired PAP • PFTs: a restrictive defect, reduced DLCO • Whole Lung Lavage
  • 146. What is the diagnosis?
  • 147. Radiation-induced Lung Injury After radiation therapy in patients with lung cancer and mediastinal lymphoma, radiologic abnormalities occur in 60-90%; 5-15% symptomatic. Radiation pneumonitis - symptoms 4-12 weeks after irradiation Radiation fibrosis - 6-12 months after irradiation • Imaging: radiographic abnormalities confined to radiation field with “straight line” effect • Management: symptomatic pneumonitis, prednisone 40-60 mg/d x 2 wk, then taper over 4-12 wk